You are on page 1of 59

Enhanced Competency Appraisal (ECA)

● ID- is responsible for a person’s survival (eating, drinking,


PRELIMS sleeping, sex)
○ Impulsive part (things that a person wants to
PSYCHIATRIC NURSING accomplish or achieve immediately)
■ Not too much thinking is done when
Anatomy and Physiology making a decision
■ Giving in to desires without control
● Frontal lobe ○ “I want pleasure”
○ Language- Broca’s area is located here ○ Non-negotiable pleasures
○ Learning ○ Works on a person’s pleasure principle
○ Personality ○ Dominant in an antisocial and narcissistic client
○ Judgment ■ Does not follow the rules or laws
● Occipital lobe ● Superego- small voice of God
○ Vision ○ “Should not”
○ Colors are processed by the optic nerve and ■ You should not eat yet, do not drink
directly travels to the occipital lobe to process the yet, do not have sex
stimulus ○ Conscience
● Temporal lobe ○ Morality principle
○ Hearing ● Ego- executive decision maker
○ Smelling ○ it balances the id and superego
● Parietal lobe ○ Known as the reality principle
○ Touch
○ Taste The ward policy: watch TV till 9pm. The antisocial client says, ``I want
to watch TV beyond 10pm.” The best response is:
Which of the following is an antianxiety?
A. "What you want is what you'll get."
● Clozaril- antipsychotic; decreases the dopamine B. “I'll discuss this with the physician."
○ Dopamine is dominant in people with schizophrenia C. "You can only watch TV till 9pm. You shouldn't watch
● Nardil- monoamine oxidase inhibitor; increases serotonin, beyond the said schedule.”
epinephrine, and norepinephrine D. Ok, If you allow me to watch TV with you.``
● Tofranil- tricyclic antidepressant; increases serotonin and
norepinephrine ID dominant people/ disorders (MAN)
● Vistaril- antianxiety (minor tranquilizer); increases GABA
● Mania, antisocial, and narcissistic disorders
(gamma aminobutyric acid)
● When a client’s id is dominant the nurse must be the
○ Inhibits the neuron’s action potential (decreases
superego
brain activity)
○ Nurses must be the opposite of the patient
Major tranquilizers ● Narcissistic people will use other people to get what they
want
● Termed as the antipsychotic agents
Mania
Connection of thoughts, beliefs, emotions, and behaviors
● Occurs to people with bipolar disorder
Beliefs→feelings→behaviors ● One week of expansive elated, irritable mood
● Impulsive emotions and behaviors cannot be controlled
● There are connection between thoughts and emotions
● They want to be the center of attention
● Our thoughts determine our beliefs and beliefs will
○ Therefore, usually wears brightly colored clothes
affect our feelings, and our feelings will determine our
and makeup
behaviors
● “We are what feel and we are what we think” Superego dominant people/ disorders (OA)
● People with anorexia nervosa has inculcated in their minds
that fat is ugly and that will lead to their behavior of ● The role of the nurse is to be the Id to balance the
undereating and withdrawn behavior superego
● People who are scared will become withdrawn and violent ● They tend to forget the Id, therefore, forgets eating,
(paranoid schizophrenia) sleeping, taking baths, accomplishing the needs for survival

Why are they different from us? Obsessive-Compulsive Disorder

● Sigmund Freud ● Obsession- thought; “my hands are dirty”


○ The person you are right now has been dictated by ● Compulsion- action; e.g., wash hands
your childhood ○ This is what the client does in order to accomplish
○ Events in our childhood have a great influence on or deal with the obsession
our adult lives, shaping our personality
○ Structure of person’s personality
Anorexia Nervosa

● The superego is dominant here


Personality structure according to Sigmund Freud
Ego unless it already affects your life, health, attitude,
and behaviors
● Problems with ego causes impaired reality perception
Ego: develops after 6 months
Schizophrenia
● The infant already distinguishes him/herself different from
● As a nurse, you have to present the reality to bring him/her the mother
back ● Already determines parts of the body that he/she can
○ Do not argue just present them the reality control
What is the factory of the id, ego, and superego? ● E.g., thumbsucks when hungry and not immediately fed

● Mothers build our id, ego, and superego Superego: 18 months- 3 years old (anal stage)
○ They stimulate the libido or the sexual energy for ● Libido is in the anus
survival ● Toilet training
○ transferred to different parts of the body ● “You should not do this/ that”
○ Wherever the libido is then that is the part of the ● Development of the superego occurs here
body that it is needed for the person to survive

Id: 0-18 months (oral stage)

● Survival
● I want to eat, sleep, drink
● A baby will cry to get the attention of the mother and will
suck the breast of the mother to extract milk
○ Libido is in the mouth

● Good mother
○ Encourages their children to help them do the right
thing during toilet training
● Bad mothers
○ Punish their children for making mistakes this
results to being obedient, clean, organized
○ But if too much strictness is present in raising a
child, superego will be dominant, obsessive-
compulsive disorder will present
● When a child cries you need to feed the infant in order to
○ Antisocial disorder if a parent is too lenient to tell
successfully address the Id
the child what is wrong and what should not be
○ The infant will feel that s/he is important and this
done
will be brought up to later ages
● An obsessive-compulsive client is anal retentive
● If an infant is ignored s/he will feel not important
○ They will not let their feces out because they want
○ When the infant grows up will show isolation from
to be clean and organized
others– narcissistic disorder will present
○ Very strong sense of conscience
○ Consistency in infant care is needed to prevent
● An antisocial client is anal expulsive
stresses to the infant
○ There should only be one caregiver at this stage 3-6 years old (phallic stage)
● Narcissistic behavior presents d/t inconsistencies in infant
care ● Libido goes to the genitals (penis and vagina)
● Fixation will present as a defense mechanism ● Close to parents
● An adult can still use the mouth as a coping mechanism, in ● Preschooler
this case, it is called regression ● Oedipus complex- little boys are more attached to the
mothers
Fixations vs. Regression ○ Little boys competes with the mothers love
○ The thinking of “I want mommy to love me, but
● Fixation means that a person is consistently stuck within mommy loves daddy” the little boy will now start
a stage and has not advanced to mimic the father— identification
○ E.g., continuous thumbsucking until 10 years old ○ Little boys have castration fear because of the
● Regression means that a person is returning to a previous competition for the mother’s love
stage ● Electra complex- little girls are more attached to their
○ E.g., thumbsucking has subsided but it returns fathers
when a person experiences stress ○ The little girl will copy the mother to achieve the
○ Return to an earlier developmental stage father’s love (identification)
○ Regression, in fact all defense mechanisms, can
be considered normal during stressful times not
○ Little girls have penis envy towards little boys ○ If a person has a good experience with what they
because they think that they have a penis that fear, it will eventually decrease
has been cut ● All energy of a child will be placed to reading, writing, and
● Children without or with fathers may have father figure arithmetic
issues ○ Instead of having sex, mag-aral ka pota ka–
○ Those without fathers may look for father figures sublimation (unacceptable urges are converted
○ Those with good relationships with their fathers to behaviors that are accepted)
may look for relationships who are similar to their
fathers 12 years old and above (genital stage)

● Choose a person you’re going to fall in love with


● People at this ages already engage in sex
● Sex is important and needed by human beings to
continuously multiply

Anxiety

● Increase in epinephrine and norepinephrine from the


adrenal medulla that is stimulated by the SNS when there
● Little girls who experience this may become magnetized
are stresses
towards abusers
● The triggering of SNS should be halted to prevent anxiety,
○ Because they equate physical, emotional pain to
therefore antianxiety agents will be given
love
○ “If I do not feel pain, then I do not love” Antianxiety agents
● Domestic violence is an intergenerational issue that should
be solved through psychotherapy ● Given to stop the anxiety from escalating
○ Kids will learn from one abusive parent that in order ● It increases the GABA (gamma amino butyric acid)
to show love you have to hurt the other person ● GABA is an inhibitory neurotransmitter - slows down body
○ To address this, explore the mindset and know how processes
this affects their lives and use it to navigate ○ Slows peristalsis, urination, salivation
through the terrain ○ Drowsiness
○ Do not dictate their mindset, let them realize it, for ○ Dry mouth
them to have a stronger grasp of it ○ Orthostatic hypotension (dangle feet before
standing up; gradual change of position)
Identification vs. Introjection ● A patient may develop dependency to antianxiety agents
○ Therefore, weaning is done to prevent tolerance
● Identification is more general
form the drug
○ Copying the entirety of a person
○ Don’t stop the drug abruptly or else, there will be
○ “Like father, like son”
rebound phenomena (return to anxiety,
○ “Like mother, like daughter”
tachycardia, tachypnea, nervousness, agitations,
● Introjection is specific
other symptoms of anxiety)
○ Specific traits or thoughts are copied
■ Gradual withdrawal is recommended
○ “Ako rin”
○ Possible seizures within one week after stopping
Projection vs. Introjection anti anxiety medications
■ Because halflife of the medication has
● Projection is putting the blame in others been consumed
● Introjection is assuming the trait of another individual
● These two defense mechanisms are the polar opposites Anxiety Disorders

Dependent Personality Disorder ● Fear is to preserve existence


● Anxiety is the vague sense of impending doom
● Activities are necessary to develop independence
● These are people who have hard time saying no What led to the development of anxiety disorders?
● Characterized by excessive need to be taken care off
● Genetics play a role in anxiety disorders
● Inability to make decisions for themselves
● Those with family members with OCD have 9 times higher
● Clouded by fear and anxiety that leads to these people
risk of also developing OCD
becoming pessimists
● They will do what others will tell them what to do, they will Neuroanatomy: active limbic system
never initiate
● There is a part of the brain that lights up when we are
6-12 years old (latency) afraid and it is known as the amygdala located in the limbic
system
● The higher the uncertainty, the higher the anxiety
○ The greater the agitation of the amygdala, the
● They find certainty and predictability to address anxiety
greater the anxiety
(rigid-OCD)
○ If there is reduction in the action of the amygdala,
○ Frustration and depression may be felt when what
anxiety will also be reduced
is planned is not met
● Intervention for school phobia- encourage them to stay and
overcome the fear
○ Gradual exposure to fears
● Sense of surrender addresses anxiety
● Sleep paralysis- awake but unable to move
○ Half asleep, half awake (partly a dream)
○ Can happen during REM sleep (rapid eye
movement)
■ Awakening at REM sleep while
dreaming
■ You are not able to distinguish what is
real and what is not
○ It's natural to have auditory hallucinations for the
first 10 minutes of sleep
○ Freud: dreams are reflective of what is happening
and what you are avoiding in your life
(symbolism)
■ Issues that you cannot confront in the
waking moment
○ Dream analysis creates insight and self-awareness

● High epinephrine and norepinephrine


● Low GABA
○ Higher levels of restlessness, anxiety, and agitation

Is this genetic, neuroanatomy, or neurotransmitter perspective?

● Hereditary
● High epinephrine and norepinephrine, low gaba
● Active limbic system

Psychological factor: change ● Upbringing is an important factor


● The psychological state, that life is filled with uncertainty, ● If parents are overprotective/over intrusive “clear the way”
is a major factor ○ Once the child discovers that life is not perfect and
fails there will be deep uncertainty about self
○ Whenever there is change, uncertainty will occur,
○ Children should overcome obstacles, learn from it
then anxiety will ensue
and become resilient
● Changes in the environment may lead to crisis
● If parents give predictable response, task assignment, and
Talk about your anxiety independence
○ Develop confidence, self-worth, self esteem
● Talking about it may help in decreasing it ○ Let them fall, for them to learn
● Brings you to a relaxed state ○ They will learn how to face obstacles and be
● As nurses, encourage patients to express their feelings independent
○ The more you express your feelings, the less ○ Perseverance is valuable
overwhelming it gets ○ Done to gain total confidence to control life aspects
○ Affect labeling- putting feelings into words
● Anxiety→ uncertainty→ darkness What events in the past may condition a person that he is not in
● You enlighten yourself control, hence, leading to anxiety?
● A support system is needed by a client with anxiety (family,
● Powerlessness over nature, perpetrator, environment
friends, healthcare providers)
● D/t traumatic events
○ This is uncontrollable

Social factors

● If a person engages in too many endeavors and


engagements there is a tendency to lose control of the
other variables
○ Multi-tasking loses track of other things
○ Too many tasks will lead to anxiety
● Worry→ hyperactive amygdala→ aggravated by past ○ So many things to do, so little time
regrets, future anxieties ● Things that we do not want to confront also creates anxiety
● To be in control of anxiety, you control your amygdala (agression, sexual fantasies)
○ 8 minutes of mindfulness a day will give the person Psychodynamic
the ability to control anxiety
○ Mindfulness means being in the present moment, ● Impulses are nearing awareness
paying attention ● Things you hide are now exposed
● Antianxiety drugs may also be taken
Behavioral
○ Taking drugs is not a weakness :c, being able to
admit that you are weak is brave <3 ● Conditioning
● Butterfly hug with deep breathings ● Observational learning
● Parents, relative, people around a child who are always ○ Decisions cannot be made by people with severe
worried will also raise a child who is also very worried most anxiety
of the time ■ The nurse should make the decision for
● To treat anxiety, you need to be aware where it came from the client
● To be able to handle patients with anxiety, you have to be ■ We become directive
aware of your anxieties first ○ Speechless
● You learned how to be anxious ● Panic (+4)
○ Your behaviors and how you react are something ○ Grave concern
you learn from your environment ○ Cannot control themselves
○ May have personality disintegration and may
Cognitive commit suicide
● Self-defeating thoughts ○ The nurse’s priority should be safety
○ “I can’t” ■ Never touch a client who is having a
○ “I can’t be successful” panic attack
○ “I can’t do anything right” ○ SNS stimulation
○ “I don’t have the money” ○ Panic attack is the highest level of anxiety
○ “I’m not smart enough” ■ Increased HR, RR
○ “Things will never change” ■ Increase RR → affects CO2 d/t
○ “What if… I fail, make a mistake… embarrass excessive blowing out →
myself” respiratory alkalosis (pH)
○ “Who you are depends on what you think” Nursing Diagnosis
● Everyday a person will experience anxiety and negative
thoughts, in order to conquer it, a person has to counter it ● Ineffective individual coping
with positive thoughts ○ Coping mechanisms are effective if they result to
something productive
Humanistic ● Powerlessness- I have lost control of my life
● Impulses nearing awareness Interventions:
● We become anxious because we cannot realize our own
potential ● Decrease anxiety
○ You have talents, skills knowledge, potentials but ● Decrease environmental stimuli (bring to a private and
you do not realize your talents quiet room)
○ It is limited, restrained, lots of prohibitions ● Relaxation techniques
● The more you hide what you can do, the more you hide ○ Affect labeling: enables control of emotion
your true self, the higher the anxiety
● Social repression of true self, the worse the anxiety will be Label your emotions
○ Afraid of rejection, stigmatized, discrimation
● The more you hide behind that facade the more anxiety
you will have

Assess: Level of anxiety

● Mild level of anxiety (+1)- alert


○ Restless but learns fast, and there is a very wide
perceptual field ● Are you angry, sad, anxious?
○ Ability to sense things around you (eyes are ● Translate them into words
clearer, hearing is sharper)
You are what you see
○ Words are enough for a mildly anxious client,
awareness of anxiety is the first step to change ● If you keep on watching sad news then you will become
○ Tell the patient “you seem restless” to help them be sad
aware ○ Decrease your exposure to sad news but still keep
○ Speaks clearly yourself informed
● Moderate level of anxiety (+2)- distracted ○ Read/ watch happy news or stories
○ Person is pacing, restless ● Relaxation therapy- means you have to change what you
○ Perceptual field is narrow, sensory organs are not hear and what you see
responding well to the stimulus presenting ○ Listen to songs that you like and look at things that
○ More distracted, stuttering, stammering you like to see
○ Levels of anxiety may be determined through ● Ineffective coping means that behaviors and thoughts
verbal cues already affect activities of daily living
○ Cannot think clearly ○ May create harm to self and others
○ Selective inattention: lacks stimulus perception ● When we are excited the epinephrine and
(sensory organs are less sensitive to the norepinephrine will increase→ excitement→ decline
surroundings) of epi and norepi→ sadness
○ As early as this period, PRN meds are already ○ It is normal to have sadness after happiness d/t
given for it not to further progress normalizing norepinephrine after its increase
■ Benzodiazepines (diazepam) ○ You cannot be excited or happy all the time, mood
● Severe level of anxiety (+3) should normally go down (normalizing of the
○ “Don’t know what to do, don’t know what to say” chemicals in the body)
Bipolar Disorder

Profile of People with Bipolar D/O

● Bipolar disorders are more common in females age 20 y.o.


and above
○ Genetics also play a role in the development of
bipolar d/o
● Those who are always stressed
● Obese

Bipolar and Related D/Os

● Mania - 1 week or more, excited, irritable, elated mood


● Hypomania - 4 days
● Delusion or false beliefs
○ “I am the queen of the world” ● Four days of abnormal mood
○ “You are my slaves, you have to bow down before ● Elated mood
me” Bipolar: Etiology
● Intense moods
○ May manifest as always shouting Biological
● Euphoria
● Genetic- monozygotic/ identical (20-75%), dizygotic/
○ A sense of psychological well being
fraternal (0-8%)
○ Feeling on top of the world
● An enlarged amygdala(located in the limbic system) is
● Hyperactivity
seen in some people with bipolar disorder
○ Anger and excitement cannot be regulated
● Decreased cerebellum (emotion regulation)
● Postpartum depression can lead to bipolar disorders
○ Responsible also for balance
○ D/t increased s/sx of depression
○ Also balances the emotions
○ Postpartum psychosis (more than 1 day less than 1
○ This is where alcohol also a takes effect (alcohol
month) may lead to bipolar d/o
intensifies emotions)
Difference between Bipolar 1 and Bipolar 2
Psychoanalytic
Bipolar 1 (severe) Bipolar 2 Cyclothymic Freud: Mania is the liberation from depression due to loss of loved
one
1 episode of mania 1 episode of major 2-year period
depression + 1 ● Hypomania- ● Mania can occur after the loss of a loved one
hypomania mild symptoms ○ Forced happiness to combat sadness
of mania ○ Suppression is a defense mechanism where you
● Dysthymia- have conscious forgetting
mild depression ○ Sadness is inflammatory :OOOOOOOOOOO
● Loss of loved one→ depression→ mania
1 week (duration of Hypomanic-depressive Behavioral
mania)
● Heightened sensitivity to reward (more pleasure from
positive event)
Manic, hypomanic, or No major depression or
depressive mania

Psychosis (not in No psychosis No psychosis


touch with reality)

Social and Social and Stable mood - less than


occupational occupational two months then repeats
impairment impairment the cycle Assessment (Mania- manic episode)
● Affects Appearance
daily life
and their ● Heavy makeup (flamboyant)
jobs ● Flamboyant, flashy, colorful clothes
● Colorful individuals (brightly-colored clothes, accessories,
and makeup)
What is Hypomania?
Behavior

● Hyperactive
● Loud voice
○ Sudden shouting
○ Easily angered (cannot control emotions)
○ Do not argue with, then just stay silent and calm,
because they will become more aggressive if you
argue with them
○ When they become violent, they may be restrained ■ Punching bags are used to transfer
○ When the bipolar person curses, tell them that it is their energy (instead of punching
inappropriate people T…T)
○ Wait for their emotions to subside before trying to ○ To divert sex and aggression
intervene ● Crisis→ depression→ mania→ major depressive
○ Bipolar people need a quiet environment disorder
○ Major depressive disorder can lead to suicide and
Maslow’s self-harm
Manic Episode Mood Stabilizer: Lithium

● Check kidneys before giving lithium


○ 95% of lithium is excreted by the kidneys
● Level- 0.6-1.2 mEq/L (or 0.5-1.2 mEq/L)
● Increase urination
○ When taking lithium there is increase in urination
○ All drugs for mental disorders cause urine
suppression, except for lithium (polyuria)
● Tremors, fine hand; takes effect after two-four weeks
○ Similar to tremors of the parkinson’s disease(but
pill-rolling tremors are specific for this disease)
○ Lithium will be taken for a lifetime
○ Injectable lithium is good for one month (this is also
given to depression with psychotic episodes)
● Hydration increased up to 3 L/day
● Because of hyperactivity (does not stay still)→ ○ D/t polyuria and to prevent lithium toxicity
undereating ● Increase Uu- increase in feces
● The person should be able to carry their food (finger foods) ● Mouth, dry
e.g., ham sandwich ○ Lithium is hyperosmolar (high osmotic pressure,
● Hyperactivity relieves tension high ability to pull water)
○ Focus on fatigue ● First signs three signs of lithium toxicity
● Excitement→ can’t sleep (they are also low in sleep) ○ Nausea, vomiting, diarrhea
○ Provide a private room that is quiet with no ○ If (+) diarrhea, rehydrate the client first, make sure
stimulus to allow them to rest and sleep that electrolytes are replaced
● Excitement→ racing thoughts/ hyperactive ○ To know if lithium related, blood test is done (done
○ When emotions are high, judgment will be impaired every month or 3 months)
○ Bipolar people may manifest as impulsive ● Sodium should be normal in order to prevent lithium
○ Bipolar may also manifest gambling disorder toxicity
● Manic clients may be seductive (↑sex) ○ (x) salt-restricted diet
○ Excitement leads to engagement in risky activities ○ The body confuses lithium as sodium, therefore,
■ E.g., engagement in unsafe sex d/t retains it
excitement that predisoposes them to ● Do not allow patient to sweat as this also expels sodium
STIs that may also lead to lithium toxicity
● Crisis→ depression→ mania ○ (x) marathons, basketball, activities that causes a
● A client with mania may be a danger to self and others person to sweat too much
○ They lack insight, hence, direct them
● Risk for injury/other-directed violence Additional Notes
○ Priority is safety ↓ serotonin → depression
● (+) intense, extreme emotions
○ Emotions fastly shifts Depression can happen without having a problem because it may
● Impaired social interaction be chemical, biological in nature
○ Frequent cursing
○ Bipolar people know what to tell a person that will Eat foods that may increase serotonin (yellow)
definitely hurt them, can easily provoke people ● Foods rich in tryptophan (serotonin is a derivative of
○ This leads to caregiver role strain and caregiver tryptophan)
fatigue as well (nursing diagnosis for the family
who takes care of the bipolar patient) Apathy is felt by suicidal patients
● High self-esteem
● Take note of the duration, frequency, and its effects to
○ T-A-S-K
your life
■ Avoid tasks that involve competition
(e.g., basketball, volleyball. pingpong) Depression may be manifested as lack of interest, low energy
■ Gardening levels
■ Wiping/sweeping the floor
■ Activities that require gross motor ● But depressed people can still be high functioning, have
skills (big movements) accomplishments but it depends on the kind of
■ Avoid activities that needs fine motor depression the patient experiences
skills (cross-stitching, scissors) ● Persistent depressive disorder→ can still have
■ Escorted walk outdoors accomplishments
○ But a person should still learn that the
accomplishments that I have will not remove
the sadness s/he has to feel
○ We should accept that we are vulnerable to
negative emotions
○ Accomplishments doesn’t make us immune to
emotions
● Major depressive disorder→ low energy, low
interest, low motor ● Enlarged ventricles→ small brain volume
● Using medications is a sign of powerfulness against the ● Family- first degree relatives→ 10x the risk
disorder ○ If you have family members who are schizophrenic,
● Valproic acid is a good substitute for lithium condition yourself to be able to create readiness
● Suicidal thoughts should not be ignored. Talk about it and prevention
and explore the suicidal thoughts ● Viral infection→ neurodevelopment
● If they feel affection and care, they can still feel ● Brain activity

What is the exact cause?

● Unknown- idiopathic
● But it was found that dopamine is increased in
schizophrenic people

Nursing Process

Assessment: content of thought

● “What are you thinking about?”


● Schizophrenic people may seem physically normal but
when you talk to them they will manifest disruption of
thought
Schizophrenia
Neurotic reality

● Still in the real world

Psychotic fantasy

● Already in the fantasy world


● “I can hear voices” even if there is no one speaking

Nursing Interventions

● Escape of people to their fantasy worlds ● Present reality


● Promote safety
Etiology
Evaluation: improved though process
Genetic
Assessment (continuation)
● Schizophrenia can be passed down
● Monozygotic twins (80%) What is the first step?
○ If one twin is (+) of schizophrenia there is 80% that
the other twin will also be schizophrenic ● Affect is the external manifestation of the mood
● Mood is the internal emotions
Stress Diathesis Model ● If these two are congruent or the same→ appropriate
affect
● Interaction between inherent vulnerability and ○ Happy and smiling
environmental stressors ● If these two are not congruent→ inappropriate affect
○ Genetics plus environment can develop ○ Happy but crying
schizophrenia ● Flat affect- no longer feels anything
● Escape real world and preferring to live in their fantasy ○ No emotions shown
world ● Blunt affect- incomplete
○ Fantasy worlds of schizophrenic people are the ○ Affect cannot be deciphered
exact opposite of their real life ● Ambivalence- being not able to choose; pulled between
two opposing forces
● Autism- isolated, withdrawn
Neuroanatomy ● Associative looseness- client has so many unrelated
ideas
○ Jumps from one topic to another without correlation
with one another

Flight of Ideas Looseness of Association


● No more positive symptoms, just withdrawn (mostly
Jump from one topic to another Repeating words
manifests negative symptoms)
I am going to the mall. Where You are the one. The one who Unclassified/Undifferentiated
is the light? Come here mineral is flying. Flying is on the floor.
water. Hurray! The floor is clean ● Mixed classification
● Cannot be classified
These two conditions arises d/t to the hyperactivity of the patient’s ● Exhibits two or more of the other types of schizophrenia
brain
In the DSM-V these subtypes have been excluded

Positive Symptoms Negative Symptoms


Clinician-rated Dimensions of Psychosis Symptom Severity
Recharged Lowbatt
● All these factors are checked for their severity
Too much thinking, talking, Withdrawn, does not talk, ● Hallucinations: sensory
activity isolated ○ Can see, hear, touch things that are not
experienced by others
Sociable Hypoactive ● Delusions: false beliefs (more on the thoughts)
○ Delusion of grandeur
○ Delusion of persecution
Based on the old classification: ● Disorganized speech
● Abnormal psychomotor behavior (waxy flexibility)
Five types of Schizophrenia
● Negative symptoms
Disorganized ● Impaired cognition (problems in memory, memory retrieval,
and judgment)
● Combination of positive and negative symptoms ● Depression
● Hebephrenic giggle ● Mania
Catatonic Antipsychotics (major tranquilizers)
● Waxy flexibility: when moved extremity does not return to ● Risperidone (1-10 mg/ day)
initial position ● Haloperidol
● Manifests negative symptoms ● Fluphenazine
○ Does not move, does not speak ● Clozaril
● Thorazine (chlorpromazine)
The brain process: ● Decreases dopamine and controls the symptoms of
schizophrenia
● Input→ process→ output
● When dopamine decreases acetylcholine will increase,
● All that a person sees during hallucinations are from the
therefore, this will cause pseudoparkinsonism (tremors)
sensory register
○ Sensory register: seen, heard, felt
○ Sensory register→ short-term memory→
long-term memory
● You can only remember things that has happened to you
or what you have seen or heard so long as you are
exposed to it
○ These are retrieved from your long-term memory

Paranoid

● Suspicious
● High risk for other-directed violence
● Mistrust→ scared→ withdrawn
● Priority is to develop trust Extrapyramidal Side Effects
● 1-on-1 interaction for a short period of time
● Frequently check on the patient and make sure when you ● Akathisia- restless, inability to sit still
say that you are going back after 10 minutes, go back after ● Akinesia- muscle rigidity; inability to control movements
that time ● Dystonia- torticollis, opisthotonus, oculogyric crisis;
○ Do what you say muscles contract involuntarily, causing repetitive or twisting
● Foods must be placed in a sealed container movements
○ As much as possible food must be prepared in
front of these patients
● Medicines must remain wrapped
● Eye contact should be done when communicating with
client with this D/O
● The client is most resistant in the orientation phase

Residual
● Neologisms
○ Creation of new words that has a meaning to the
patient, but in reality, it does not have
meanings
○ e.g., plungplang, priskip, sertugil
● Delusion: persecutory
○ The NBI is out to get me
● Delusion: grandeur
○ I’m the queen of the world

○ Torticollis (wry neck)- a twisting of the neck that Parkinson’s Disease


causes the head to rotate and tilt at an odd angle
○ Opisthotonus- back becomes extremely arched ● Acetylcholine is the on switch of the muscles
due to muscle spasms; arched back ● Dopamine is the off switch of the muscles
○ Oculogyric crisis- spasmodic movements of the ○ Dopamine is low and acetylcholine is always high
eyeballs into a fixed position; fixed stare in this disease
● Tardive Dyskinesia- irreversible EPS ● “You’re always ON, with parkinsON”
○ Lip smacking
Hallucination & Illusion
○ Tongue protruding
○ Cheeks puffing ● Nursing diagnosis for these two are disturbed sensory
● Neuroleptic malignant syndrome perception
○ Fever + muscle rigidity

Tremors are to be expected when taking antipsychotics, Hallucination Illusion


therefore, these two classes of medications are given to
Stimulus Absent Present but
counteract tremors: processing of the
● In order to decrease the EPS, anticholinergic stimulus is incorrect
medications (triggers the SNS) as PRN medications will
Voices are heard, or A person saw a
be given to decrease acetylcholine and decrease muscle things are seen but rope but interpreted
contractions things perceived are it as a snake
○ Akineton not really seen
○ Artane
○ Benadryl ● When there are auditory hallucinations, you have to ask
○ Cogentin what are the voices saying
● Dopaminergic is given to increase the dopamine and ○ To determine if there are command
reduce EPSE and muscle contractions hallucinations, these are voices that directl the
○ Levodopa client to perform an action
○ Larodopa ○ “The voices said that I would stab myself later”
○ This will guide you on what to do
Other Side Effects of Antipsychotic Medications ● Tell the client, “I know the voices are real to you, but I don’t
hear them” and reassure the patient that s/he is safe
● Photosensitivity- extreme sensitivity to ultraviolet rays ○ What these patients perceive are real to them,
from the sun and other light sources cardinal rule “do not argue with the client, just
○ Severe sunburn present reality”
○ Sunscreen, SPF 30 ○ Diversional activities are also effective to prevent
○ Sleeves, long the brain from creating the voices a patient hears
○ Sombrero, wide brimmed hat
● Agranulocytosis (clozaril) Depression
○ Low WBC count
○ First sign is sore throat ● A common mental disorder with more than 300 million
people of all ages
● Leading cause of disability worldwide, with 800,000
Additional s/sx of Schizophrenia
people who die due to suicide
● 1 person commits suicide every 40 seconds
● Magical Thinking
● People with depression have impaired judgment
○ “I can turn you into a frog”
● PostPartum Depression is experienced by mothers after
● Echolalia
giving birth
○ Repeating what the other person says
● Echopraxia Antidepressants
○ Repeating what the other person does
● Word salad ● Fluoxetine
○ Speaking of words that do not rhyme ○ Capsule 20 mg
○ e.g., mute, beat, lice ● Sertraline
● Clang Association ○ Scored tab, 50 mg
○ Rhyming words ● Amitriptyline
○ e.g., life, wife, knife ○ Tab. 25 mg
○ THIS BITCH CAUSES THE URINE TO TURN
○ Perception
BLUE
● What do you do about the problem?
● Escitalopram
○ Coping
● Paroxetine
● Support- people
● Mirtazapine
● This is done for the person stay afloat for the next six
● Venlafaxine
weeks
● Duloxetine
● Vortioxetine Positivity Ratio
Assessment ● Think of 3 things that make you happy for every one
thing that make you sad
Signs and Symptoms of Depression

● A slowing down of thought


○ Or if depressed people think they usually think of
the negative Nursing Diagnosis
● Diminished interest or pleasure in activities (anhedonia) ● Risk for self-directed violence
○ Hedon means happy or pleasure ● Self-mutilation
○ Decreased interest in things you usually love to do ○ If already hurt self
● Depressed mood
● Fatigue or loss of energy Planning/Intervention
○ They should be given with tasks/exercises to
● Stay
increase endorphin
● Speak
○ This will also preoccupy the mind
○ Allow the client to talk about it
● Feelings of worthlessness or excessive inappropriate guilt
○ Directly ask the patient, “do you plan to die by
● Significant weight loss or gain
suicide”
○ Some people who are depressed tend to undereat
○ Because you cannot make assumptions
○ But there are also some people who tend to
● Safety
overeat
○ Remove sharp objects
● Sleep disturbance (insomnia or hypersomnia)
● Listening is the most important skill
○ An overactive mind prevents sleep
● Tell the client, “I will be here”
○ Thinking of negative thoughts
● Never tell the client “don’t worry, be happy”
● Suicidal ideation (recurrent thoughts of death, recurrent
● Do not tell the client to shake it off
suicidal ideation without a specific plan, or a suicide
attempt or specifin plan for comitting suicide) Evaluation

Grief Counseling ● Patient should always be safe

● What/ how are you feeling? Serotonin


● What other things have you lost? ● The chemical that makes us happy
○ other aspects of the person you have lost ● A low serotonin may result to depression, therefore,
○ e.g., confidant antidepressants will be taken
● Were there things you haven’t said? What are these?
● How would you rate from 1-10 your relationship with this Antidepressants
person?
● If you die, how would you want your family or the people SSRIs (selective serotonin reuptake inhibitors)
you have left to feel? ● Safest
● Suppressing and not talking about your grief is okay but ● SSRI only increase serotonin
letting it out and telling people important to you about ● Low side effects
them also helps because this decreases the burden ● Fast effects, within 1-4 weeks
○ As you repeat and talk more about it, the ● Escitalopram, prozac, zoloft
heaviness it brings becomes light
● We do not move on with our loss or with the loss of our Tricyclic Antidepressants
loved ones, but we learn to deal with it through time
● Two-four weeks before it takes effect
(time heals all wounds)
● Increases serotonin and norepinephrine
Grieving Process ● Higher incidence of side effects (because it increases more
hormones and neurotransmitters)
DABDA ● Tofranil, elavil
● Denial→ anger→ bargaining→ depression→ Common Side Effects of Antidepressants
acceptance
● When a person arrives at depression, it’s either a person ● Constipation
will progress to acceptance or to suicide ● Tachycardia
○ Support system is needed here to prevent ● Urine retention
suicide ● Dry mouth

Crisis Intervention (done for 4-6 weeks) Monoamine Oxidase Inhibitors (MAOI)

● You the person how s/he see the problem? ● Monoamine oxidase is involved in the breakdown of
norepinephrine, epinephrine, serotonin, and dopamine
● Takes effect at 2-6 weeks Legal Responsibilities of Nurses
● Avoid tyramine-rich foods to avoid hypertensive crisis
● Last to be prescribed because it has the highest incidence ● Informed Consent
of side effects ● Delegation
● Drugs and Prescriptions
Tyramine-rich foods ● Documentation- charting

● Avocado Profession
● Aged cheese
● Beer (fave po ni stacwi 2, tru arat na) ● "a calling in which it's members profess to have acquired a
● Chocolate specialized body of knowledge and skills acquired through
● Fermented foods training or by experience so that they may guide others in
● Pickles this special field"
● Preserved foods ● Nursing is a profession
● Soy sauce 1. Because it is a calling (service-oriented)
Summary of the Most Common Mental D/O 2. To be a member of the nursing profession you
have to go through licensure and registration
○ After passing the board exam you are
Anxiety D/O
already a licensed nurse but before you
● ↑Epinephrine and norepinephrine, ↓ GABA
● Antianxiety agents (valium, librium, ativan) can practice you have to be registered
● Focus: to decrease anxiety and increase relaxation and it you can only be registered after
● Mild, moderate, severe, and panic oath taking
● NDx: Ineffective individual coping 3. Necessary competencies should be acquired
● Goal: Prioritize safety through training
4. Patient-centered or client-focused
Schizophrenia ● The ANA formulated the definition nursing as a profession
● ↑Dopamine
● Antipsychotic agents (haloperidol/ haldol, risperidone, If you pass a counterfeit copy of your TOR in the PRC and still
clozaril, thorazine, and fluphenazine pass the board exam, PRC will not be allowing you to take the
● Antiparkinson: Anticholinergic and dopaminergic oath and you will be disqualified. (grounds for disqualification)
● Focus is the symptoms
● NDx: disturbed sensory perception and disturbed thought ● AAACCESS (characteristics of why nursing is a
pattern profession)
● Goal is to present reality ○ A- accountability/ liability- being liable for your
professional actions
Bipolar ■ If a nurse fails to perform a task or
● ↑Norepinephrine makes mistakes criminal,
● Lithium
administrative, and civil cases will be
● Maslow’s (food,
sued
water, basic needs,
sleeping,
hyperactivity, sexual Administrative Civil Criminal
urges) because
patients tend to Where is it
forget these d/t dealt with
hyperactivity
● Safety is priority Person who
● Social interaction is complained
not good
Penalties Suspension of Paying of Imprisonment
Depression not more than damages
● ↓Serotonin four years or
● SSRI (prozac, lexapro) revocation of
● TCA (tofranil, elavil) license
● MAOI (marplan, nardil, and parnate) ○ A- autonomy or professional independence
● Self-directed violence ■ Performs actions even without the
● Goal is to prioritize safety orders of a physician
○ A- altruism or selfless attitude
■ When there are dangers always put the
patient first
○ C- caring role or caring profession
○ C- competency, a person should have necessary
training or skills to perform procedures
■ In-service training- If the hospital
provides free training, seminars to
develop the skills of their staff
PROFESSIONAL ADJUSTMENT ■ Continuing professional development
AND NURSING (CPD)- If the person pays for their own
competency training, CPD units is
JURISPRUDENCE
already mandatory (50 hours or 15 ○ Nurse educator
units or CPD training) ● Access of patients to health care services
■ License should be renewed on or ● Nursing process
before your birthday every three years ● Charting
○ E- ethics, nursing has its own code of ethics ○ Recording concerns
■ Used when confronted with ethical ● Educating and training students
dilemma (clash between the patients ○ Even if you are not a CI so long as there are
choice and the duty of the nurse) students affiliated in the ward where you are
○ S- service-oriented (not for money daw weh) assigned
○ S- pecialized body of skills ● Supervision of subordinates
○ This includes delegation of tasks to NAs
Nursing is best defined as a profession? ○ Accountability to the subordinates
a. Concerned with nursing diagnosis and treatment ● COllaboration of patient’s care
b. Help people to do activities that contribute to health ○ To be able to provide comprehensive and holistic
recovery and peaceful death only care to the patient
c. Care for people sick or well in all health care setting ○ Four types of patient-care collaboration:
d. Assist people to self care ○ Endorsing clients to people involved in the care of
the patient
Rationale: because it is one of the reasons as to why nursing is a ○ Intraprofessional collaboration- collaboration of
profession patient-care inside the same disciple (e.g.,
morning shift nurse to afternoon shift nurse)
Which among the following is the most important attribute of
○ Interprofessional collaboration- collaboration of
professional nursing?
different disciples (e.g., the nurse asks for a new
a. Community oriented dose of insulin to the physician)
b. Observe ethical practice ○ Interdepartmental collaboration- collaboration of
c. Demonstrate self control different departments (e.g., a patient for the ICU
d. Provide human resource needs to be transported to a pulmo ward)
○ Interagency/inter institutional collaboration- e.g., a
Safe nursing practice requires a nurse? patient needs referral to social institution
● Execution of valid doctor’s order
a. Completion of BSN degree program
○ Malpractice - exceeded the scope of nursing
b. Registration before PRC
practice
c. Professional competence
○ Negligence- mistakes done within the scope of
d. Membership before PNA
practice
The most important specific characteristic of a true professional
Among the core competencies of a nurse, which is considered as a
nurse is?
dependent task?
a. Observe professionalism
a. Health teaching to post op clients
b. Sensitive to need of others
b. Administering drug orders
c. Accountable for his action
c. Formulation of a nursing process
d. Respect for life
d. Referral system to community resources
Which of the following is the most distinguishing characteristic of a
Institutional Based Nursing
professional nurse?
● Hospital-based
a. Respect rights of a co-worker
● Functions are supervised/delegated
b. Orientation towards service
● Acquire different areas of specialization
c. Care for the needs of hospitalized client
● Coordination with other HCTs (health care teams)
d. Attitude of helpfulness
Community Health Nursing/ Public Health Nursing
What is the most important contribution of nursing in professional
practice? ● Promotive of public health and Prevention of diseases
○ Because not all people in the community are sick,
a. Respect for life
therefore, promotion of health and prevention of
b. Humanistic view of life
occurrence of illness is priority
c. Spiritual health of patient
● Physical and social adjustments to clients from various
d. High value of service to fellowmen
cultures
It is simply defined as the legal liability of the nurse in the conducting ● Participate with all members of the community and health
or performing professional task? care

a. Non-maleficence
b. Autonomy
c. Accountability Laws in Community Health Nursing
d. Liability
Preventive
Professional Duties of a Nurse Under RA 9173
PD 996: EPI (expanded program on immunization)
● Care in all HCs
○ Institutional nursing ● Compulsory immunization of children below 8 years old
● Health education
○ Care for sick/ injured laborers
against common childhood diseases
○ Referral services
● Wednesday is vaccination day
■ Remove the sick person from the rest
● MMR, pentavalent, rotavirus, OPV, BCG
of the people within the company
● Can prevent TB, pott's disease (TB of the spine,
○ Home Visits and follow ups
extrapulmonary TB), leprosy, mumps (parotitis),
● Promotive/ preventive
measles, rubella (german measles), rubeola, pertussis
○ Proper Nutrition
(whooping cough), hepatitis, and polio
○ Safety and sanitation in the workplace
RA 9288 Newborn Screening Act ○ Counseling on ORDI (occupational related
diseases and injuries) or occupational hazards
● Requires mandatory screening among newborn babies ■ E.g., lead poisoning
against six congenital disorders
○ Congenital hypothyroidism As an occupational health nurse in the industry, you are primarily
○ Congenital adrenal hyperplasia responsible to?
○ Phenylketonuria (PKU)
a. Institute health measures in preventing accidents
○ G6PD (glucose-6-phosphate dehydrogenase)
(gloves, helmets)
○ Galactosemia
○ Safety should always be the top priority
○ Maple syrup urine disease- has the highest
b. Assess the employees for respiratory infections
prevalence among the six
c. Follow-up existing medical problems and appropriate
■ Urine smells like a syrup
medical referrals
■ Disease of the kidneys
d. Health education and dissemination regarding
● When is newborn screening done?
communicable disease
○ If the newborn is health, within the 24 hours of
life Nurse Educator/ Clinical Instructor
○ If inside the NICU, within the next 7 days of life
● Qualifications to be a nurse educator in the Philippines
Promotive ○ AMOR
○ Accredited nursing organization (PNA member)
PD 825: Environmental Sanitation Law of the Philippines
○ MAN/MSN in nursing or other allied health course
● Applicable outside or external environments ○ One year clinical experience
● Anti-littering campaign ○ Registered nurse
● Waste segregation program
● Anti-pollution campaign Philippine Nursing Association

PD 856: Sanitation Code of the Philippines ● The only accredited professional organization of nursing
in the Philippines
● Applicable inside institutions/establishments that offer ● PNA– the only APO in the Philippines, was founded in
food and water supply (restaurants, fast food chains, September 1922
water stations, hotels, carenderias) ● The founder of PNA is Anastacia Giron Tupas
● Sanitary permits are needed before institutions can Purposes of PNA (PUPAE)
operate ● The only Professional organization of nurses in the
● Mandatory sanitation inspections Philippines
● All establishments that offers commercial sex workers ● Unity among nurses in the Philippines
needs to have mandatory STD testing (gonorrhea and ● Professional Advancement in the Philippines
syphillis) ○ Trainings and seminars for professional
○ Gonorrhea- every two weeks growth and advancement of nurses are
○ Syphilis- every six weeks provided
● Enhancement of camaraderie among nurses
Proclamation order 539
● Declares that the last week of October as the official
Which of the following is not a function of a PHN? nurses week in the country

a. Collaborates client's care


b. Works with the community members One of the expectations is for nurses to join professional associations
c. None of these primarily because it serves the following purpose?
d. Implements primary health care
a. Promotes advancement and professional growth
among member
b. Works for raising of funds for nurses benefits
c. Facilitates and establishes new acquaintances among
nurses
d. Assist them in securing jobs abroad
Occupational Health Nursing/ Industrial Nursing
Requirements to be passed before taking the licensure exam
● RA 11058 Occupations Health and Safety Act (OHSA) ● Birth certificate
○ Requires that all high risk companies and offices ● TOR (transcript of records)
should have a company nurse (factories, ● If there are typographical errors or differences in some
chemical companies) letters between the two– birth certificate and TOR,, you
● Curative/ rehabilitative can still take the board exam just file an affidavit of
○ (+) Injury or illness within the factory or company discrepancy
○ Nurses assist and witness (during signing, the
● If a student uses the surname of the father in school that
nurse should witness)
is reflected in the TOR, but in the birth certificate
surname of the mother is used, a hearing will be ● Understood by patient
conducted. ○ Avoid the use of medical terms
● If the surname is foreign, passport is accepted ● Matured/Capacitated
○ Physical
■ 18 years old and above
LEGAL RESPONSIBILITIES OF NURSES ○ Mental
■ Anything that will affect the LOC
● Duties of a nurse (mentally incapacitated client should
not sign the consent)
Informed Consent
■ Sedatives alter the LOC - if this
● The person to obtain the informed consent is the person happens and the client has not yet
that will do the procedure (surgeon) or the physician signed the informed consent, call the
● Informed consent is a part of autonomy attention of the surgeon for a possible
○ Decision making of the patient is practiced cancellation of the surgery
● Should be with the patient before going in the OR ■ Parents/relatives only sign if the client
● Consent form is the paper itself is a minor or mentally incapacitated
● Informed consent is a procedure of informing/explaining to ○ In the absence of one or both, the consent is
the patient invalid
● Autonomy: the patient decides
Substitue/Proxy Consent

If minor or psychiatric client (in chronological order)

● Parents
● Relatives/Guardians
● Next of Kin (next in line relative; relatives nearest to the
client)
● E.R. - M.D. (The doctor’s order/discretion is accepted
during emergencies)
● Paternalism: a bioethical principle where another person
decides for the patient

If adult client (in chronological order)

● Spouse - determine status (single/married)


● Children
● Parents
● Relatives/Guardians
● Next of kin
● Doctor
● Paternalism
● Signatories are important
● The patient should only be the one to sign the consent form In any research study where individual persons are involved, it is
○ In tubal ligation or vasectomy procedures, the important that an informed consent for the study is obtained. The
couple should be present during the explanation following are essential information about the consent that you
disclose to the prospective subjects, EXCEPT:
Five Characteristics of Informed Consent
a. Consent to incomplete disclosure
● Voluntary consent b. Description of benefits, risks and discomforts
○ There should be no coercion/threat c. Explanation of procedure
○ Assault: mental fear/threat d. Assurance of anonymity and confidentiality
○ Battery: physical introduction of force (e.g.,
straints are applied) Rationale: there must be complete information
● Opportunity to ask questions For involuntary clients, informed consent is usually provided by?
○ Give the patient opportunity to ask questions
○ They should not sign the consent if they still have a. Relatives
questions b. Physician
○ Encourage the patient to ask their questions to the c. Nurse
surgeon/physician d. Chief nurse
● Treatment or surgery must be explained
○ The treatment or surgery should be explained first *Involuntary clients are admitted to the hospital without their consent
to the patient A thumb mark as a means to give consent from an unconscious
○ The surgeon explains the surgical procedure client is considered as?
(risk, benefit, amount, outcome)
○ The physician explains the medical procedure a. Valid
○ Whoever will perform the procedure, wll be the one b. Legal
to explain and obtain the formed consent c. Inappropriate
d. Invalid
Thumb mark with one witness is valid especially for illiterate clients nurse in the ward gave the wrong medication and the client suffered
paralysis, who shall be accountable?
A 17-year-old client is scheduled for surgery. The nurse taking care
of the client realizes that consent has not been signed after a. The staff who gave the wrong drug
preoperative medications were given. What should the nurse do? b. The staff nurse and the unit manager
c. The unit manager only being the superior authority
a. Call the surgeon (for possible cancellation of the d. The staff, unit manager and hospital
procedure)
b. Ask the guardian to sign the consent Delegation Procedures
c. Obtain a consent from the client as soon as possible
d. Get a verbal consent from the parents of the client

There is no emancipated minor in the Philippines

DELEGATION

● A person higher in the hierarchy assigning tasks to


subordinates
● Head nurse The following tasks can be safely delegated by a nurse to a non
○ Does not perform bedside care but can delegate it nurse health worker, EXCEPT:
to staff nurses
a. Transfer a client from bed to chair
Principles of Delegation b. Change IV infusion
● Accountability is shared c. Monitor urine output
○ The delegator will be liable for the person whom d. Vital signs
the task is delegated Which among the following tasks a RN can safely delegate to her
○ The head nurse will be accountable under aide?
respondeat superior (command responsibility)
■ Let the master answer a. A client that requires round the clock pain medication.
○ The person who performed the task will be held b. A client requiring frequent ambulation.
liable to the patient (negligence) c. A client who is under vest restraint.
● Responsibility is kept d. A client with diabetes mellitus requiring daily insulin and
● Authority is given reinforcement of dietary
When Mrs. Guevarra, a nurse, delegates aspects of the clients care Which among the following tasks cannot be delegated by the nurse
to the nurse-aide who is unlicensed staff, Mrs. Guevarra: to any ancillary health workers in her unit?
a. Makes the assignment to teach the staff member a. Vital signs
b. Is assigning responsibility but not accountability for b. Shampooing
those tasks c. referral of proper diet with the nutritionist
c. Does not have to supervise or evaluate the aide d. obtaining equipments from stockroom
d. Must know how to perform tasks delegated
Which of the following tasks could the registered nurse safely assign
Wilfred, 30 years old malo, was brought to the hospital due to injuries to an Aide? The registered nurse is planning to delegate tasks to a
sustained from vehicular accident. While being transported to the x- nursing aide.
cay department, the straps accidentally broke and the client fell to the
floor hitting his head. In this situation, the nurse is: a. Monitor the l&O of a comatose toddler client with salicylate
poisoning
a. Not responsible because of the doctrine of respondeat b. Perform a complete bed bath on a 2-year-old with multiple
superior injuries from a serious fall
b. Free from any negligence that caused harm to the patient c. Check the IV of a preschooler with Kawasaki disease
c. Liable along with the employer for the use of defective d. Give an oatmeal bath to an infant with eczema
equipment that harmed the client
d. Totally responsible for the negligence Legal Responsibility of Medication, Drugs, and Prescriptions

If ● Only registered medical, dental, and veterinary


a practitioners are authorized to prescribe drugs
● Three informations:
○ Physicians:
■ Check the name of the physician
■ Address or location of the clinic or
hospital
■ PRC license number
○ Patients:
■ Name, age, sex/gender
○ Rx:
■ Name of the drug
■ RA 6675: Generic Act
■ Generic name (brand name)
■ Brand names only are not allowed, to
Photocopying, reading the patient’s chart is not allowed without
allow the patient to buy drugs at a
the permission of the patient
lower cost
● 10 Rs of medication The owner of the patient’s chart is the hospital/institution
○ Right drug
○ Right patient The safekeeper of medical records is the Record section office
■ Most acceptable way of identifying the ● Legal documents are kept for a maximum of 5 years
client: Identification bracelet/wrist band ● Still kept because it may serve as legal evidences
■ Asking the patient’s name
■ Ask the mother/father/relative at side Subpoena- order coming from the court
○ Right dose ● Subpoena duces tecum- papers, records, documents,
charts are requested by the court
○ Right route
● Subpoena ad testificandum- a person to testify at court
○ Right time and frequency (witness) is requested
○ Right documentation
○ Right history and assessment
○ Right education and information
Dos in Charting
○ Right drug-to-drug interaction and evaluation
○ Right to refuse ● F- full, factual, and accurate
● Verbal or Telephone ○ Write objective data, do not add anything else
○ Only allowed during emergencies ○ Verbatimly write subjective data
○ Should always be in writing ● L- legible
○ Always write the date, time, name of the physician, ● I- immediately
and the complete drug order ● P- personal
○ Repeat the orders during the call for the validation
and clarification Styles of Charting
○ Within the next 24 hours obtain the signature of the ● Narrative
doctor (countersign) ● Problem-oriented medical record
● Doubts or errors ○ Focus (FDAR)
○ Call the attention of the physician, let the physician
correct and clarify themselves Narrative
In extreme situations and when no other resident or intern is
available, should a nurse receive telephone orders, the order has to
be correctly written and signed by the physician within:

a. 48 hours
b. 36 hours
c. 12 hours
d. 24 hours

Nurse Suzie is administering 12:00pm medication in ward 4. Two


patients have to receive Lanoxin. What should Nurse Suzie do when
one of the clients does not have a readable identification band?

a. Ask the client if she is Mrs. Santos


● Chronological document all activities done to the patient
b. Ask the client his name
throughout the shift
c. Ask the roommate if the client is Mrs. Santos
○ Document even the patient do not have problems
d. Compare the ID band with the bed tag
● Baseline chartered qshift
One way of verifying that the right message or order from the doctor ● Lengthy and time-consuming
was communicated effectively through telephone is by?
Problem-oriented Medical Record
a. Phrasing intelligently
● The only time that the nurse has to document is when there
b. Repeating the order/message
are presence of problems, unusual assessments, and
c. Documenting immediately even with doubts
complains from the patient
d. Speaking distinctly using enough volume
Focus Charting
Documentation
● Data - subjective or objective that supports the focus
● Charting/ recording
(concern)
Purposes ● Action - nursing intervention
● Response - patient response to intervention
● C - communication
● L - legal document Date and time Focus Progress notes
● E - evidence
● A - assurance of continuous care
● R - research
● S - statistics
Legible/readable
● The patient’s chart is a legal evidence, therefore, it should
QD- every day
be readable
q2h every 2 hours
● Documentation should be done immediately after the qAM- every morning
procedure qPM- every afternoon
● If late entry, place “addendum” qod every other day
○ Additional late documentation rtc- round the clock
KVO- keep vein open
Personal/ Confidential C./C- chief complaint
● Do not allow other people to sign for you in the charts DNR- do not resuscitate
SOB- shortness of breath
● Correct way of signing- complete name with RN
DOA- dead on arrival
Don’ts in Charting DNI- do not intubate
DNT- do not transfuse
● L- Language, jargons or words which are unacceptable DNR- do not resuscitate
○ Avoid unofficial languages
○ Should only use jargons accepted in the institution
● I - Improper corrections Which of the following is not a standard abbreviation used for
● S- spaces and skips documenting patient care?
● A- abbreviation
a. PRN
b. OD
TID- thrice a day
BID- twice a day c. NNO
OD- once a day d. NPO
OS- left eye Improper corrections
OU- both eyes
PRN- as needed, necessary ● Draw one or two straight less across the word then write
STAT- immediately now the word “error” or “mistake”
C- with ○ Dated and signed by the nurse
S- without ● Avoid leaving Spaces and skips
AC- before meals
○ Place lines on spaces and skips
PC- after meals
VSS- stable vital signs Abbreviation
htn- hypertension
AAO- awake, alery, and oriented ● Use official abbreviation
amb - ambulatory
BM - bowel movement The patient's medical record is the best evidence of the care that is
BMR- basal metabolic rate given to the patient. It is the property of?
BPM- beats per minute
BS- breath sounds a. patient owns the record while inside the hospital
Bx- breast biopsy b. The physical property of the hospital
CXR - chest x ray c. The property of the health team
DAT- diet as tolerated d. The doctor owns it
Dx- diagnostic
Fu- followup Who among the following cannot have access to a patient's record
HA- headache after laryngectomy at all times?
I&O- intake and output
a. Speech therapist
oob- out of bed
nkda- no known drug allergy b. Physical therapist
pr - pulse rate c. Lawyer of the family
brp- bathroom privileges d. Patient
Rx- treatment
sl- sublingual
sx- symptoms The nurse enters data on a chart and discovers she has written on
TO- telephone order the wrong chart. How is this error best corrected?
TPN- total parenteral nutrition
VO- verbal order a. White out the wrong information and write over it
WO- written order b. Recopy the page with error so chart will be neat
WOP- without pain c. Draw a straight line through the error, initial and dated.
AKA- above the knee amputation d. Obliterate the error so it will not be confusing
bka - below the knee amputation
a/g ratio- albumin globulin ratio
fx- fracture
hs- hours of sleep
CS- cesarean section
C/S- culture and sensitivity Ethics
KCl - potassium chloride
● Ethical dilemma- when the beliefs of the patient do not
PERRLA- pupils are round, reactive to light and accommodation
PMI - point of maximum impulse align with the treatment
UA- urinalysis Autonomy
SQ- subcutaneous
Q- every ● Let the patient decide, there should be no coercion
● Right of the patient to decide for himself without the ● E.g., a mother with cancer will choose between
influence of others chemotherapy and continuing pregnancy
1. Included here is the informed consent (right to informed
consent) Beneficence
2. The patient can always refuse (right to refuse) ● Doing good for the benefit of the patient
● If the patient refuses: ● Utilitarianism
○ Look at the consequences of the ○ Doing good based on the end-result of the actions
refusal, if it does not have any risks, (duty-oriented)
respect the decision of the patient ○ The end justifies the means
○ But if there is a risk involved, explain ● Deontology
the risk to the patient and the family ○ Doing good based on the law/rules/policies
○ If the patient refuses still after knowing ○ Law-oriented
the risks, let the patient sign a waiver
(the doctor will obtain the signature) Non-maleficence
○ If the patient wants to go home or
● Do no harm
transfer to another hospital, let the
● Avoiding physical harm/injuries
patient sign a HAMA/ DAMA form
● E.g., putting the side rails up to avoid accidental fall
Advanced Directives
Fidelity
● Tells the healthcare providers what treatment you want and
● Loyalty to the needs of the patient
do not want
● Hindi mo na trabaho ginagawa mo pa
● It also tells the healthcare providers on who you want to
● Providing extra services to meet the needs of the patient
speak in your behalf
● Direction in advance Inviolability of Life
● Written advance instructions of patient
● Principle of proper respect of human life
Two Types of Advanced Directives ● Promotion of life
● E.g., do not promote euthanasia, suicide, or abortion
● Living will
● The nurse can refuse to take part in these procedures if it
● Durable power of attorney
does not align with the moral beliefs
○ This document appoints a person to decide for the
● Active and passive euthanasia
patient in case s/he becomes unable
○ Active- active measures to cause death to the
○ No decision yet, but appoints a person to be a
patient (e.g., KCl administration)
proxy to decide for the patient
○ Passive- Did not employ any methods for the death
of the patient (stopping feedings)
Upon admission, the nurse is responsible to tell the patient that
s/he is allowed to sign a DNR form Justice

Even if the advanced directives are signed in another country, it ● Equal access to healthcare among all patients
will still be valid ● Provide fair treatment and provision of justifiable allotment
of resources
There is no expiration date for advanced directives ● Prioritization of patients depending on the urgence of
needs- triaging
The patient should be the one to sign or unless it will be deemed
invalid Triaging

Verbal directives are not allowed it should be written ● Red- emergent cases, requires immediate medical
attention and cannot wait
○ CPR
Paternalism ○ Severe respiratory distress
○ major trauma and burns of the head, face, neck
● Another person decides for the patient ○ Chest pains
○ Chemicals in the eye
Veracity
○ Uncontrollable bleeding
● Truth telling ○ Coma
● The patient has the right to be informed of truthful ● Yellow- urgent cases, can wait but not more than 60
information minutes upon admission
● But do not confirm anything to the patient (diagnosis, ○ Abdominal pain
prognosis, explanation of procedures) the doctor should be ○ Fracture
doing this ○ Major laceration
○ Refer the client to the physician ○ Renal calculi
○ Asthma with no respiratory distress
Double Effect ● Green- non-urgent cases, can wait even more than 60
minutes
● If a patient is made to choose between two equal dangers,
○ Rashes
let the patient choose the choice that will bring about the
○ Headache
good result
○ Minor lacerations
● The action must be morally good, this is where harm is
○ Cold symptoms
permissible
○ Sprain
○ Fever
● Black or none- deadT or have the least chances of survival
○ No chances of survival d. A mother with a 5-year-old boy who says her son has been
○ Pulselessness/ breathlessness complaining of nausea and vomited once since noon
○ Support measure Stewardship
○ Pain medications only till death
● Taking care of those entrusted to you
The E.R. nurse should give priority to which of the following patients
assigned to her? Totality Principle

a. A post- CVA patient who only needs discharge health ● The principle of totality states that all decisions in
teaching medical ethics must prioritize the good of the entire
b. A 17 year old girl who had an asthmatic attack 2 days ago person, including physician, physiological, and
c. A 20 year old patient admitted after a vehicular spiritual factors
accident and has head injury (assess for increased ICP)
Confidentiality
d. A 5 year old girl who has koplik spots and high grade fever
(fever is not an emergency case) ● Every information that you will know or see about the
patient should not be disclosed to other people not involved
Techniques in Triaging in the care of the patient
1. Use the ABC’s of providing care for patients and ● Exception to confidentiality
prioritization ○ PICCS
● D- disability ○ P- patient consented and signed in writing
2. determine the stability of patients ○ I- inform other members of the HCT involved in the
● For discharge
care
● Comfort measures only or ADL
● Slight deviation in the vital signs or laboratory results ○ C- communicable diseases (under the RA 11332)
● Requires health teaching
● Admitted for some day already RA 11332: The law on reportable communicable diseases
● Do not prioritize if care can wait ● All communicable diseases that reaches the epidemic
3. Prioritize emergency cases (e.g., chest pain, respiratory level
distress) ● CoVID-19
○ C- crimes (child abuse, rape/ sexual abuse, elderly
abuse)
After an endorsement, you are assigned to care for the following.
Which client do you assess first? Follow the SRR rule:
● Safety of the client, if no safety is in the choices look for
a. A 68 year old client on a ventilator who needs a sterile
words that implies safety (stay with the patient, provide
sputum specimen sent to laboratory
emotional support, silence).
b. A 57 year old client with COPD and pulse oximetry reading ● Recording (record all the assessment findings as this
from a previous shift of 90% saturation (low but within may be used as future evidence in court.)
acceptable limit) ○ Burns
c. A 72 year old client with pneumonia who needs to be ○ Bruises
started on IV antibiotics ○ Lacerated hymen, blood in the underwear
d. A 51 year old client with asthma complaining of SOB ○ The child is complaining about the father
after using bronchodilator inhaler (medications are ○ Never discard any clothe, article in the body of
already given but still has difficulty of breathing) the patient. Keep it safe in a ziplock. (for DNA
testing)
Being the new staff in your ward, which among the following patients ● Reporting
you need to prioritize? ○ Under the VAWC law, report the incident
whether the parents of the patient want to or
a. Mr. X, with a temperature elevation to 99 F after morning not within 24-48 hours (to the barangay)
vital sign ● Refer to services (DSWD)
b. Mr. B, having chest pain episode occurring during ○ To keep the patient safe if the perpetrator is
morning care within the home.
c. Mr. V, BP increase of 10 mm Hg after morning exercise
d. Mr. L. heart rate increase of 10 beats per minute after ○ S- safety is at risk
ambulation

On the evening shift, the nurse evaluates several clients who were
brought to the emergency department. Which in the following clients
should receive highest priority?
It is unethical to tell one's friends and family members data about
a. an elderly woman complaining of a loss of appetite and patients because doing so is a violation of patient's right to:
fatigue for the past week
b. A football player limping and complaining of pain and a. Least restrictive environment
swelling in the right ankle b. Informed consent
c. A 50-year-old man, diaphoretic and complaining of c. Confidentiality
severe chest pain radiating to his jaw d. Paternalism

Pain of angina is localized, MI pain already radiates


As standards of ethics, this represents an understanding and ● Omission or commission of an act that lead to harm that is
agreement to respect another person's right to decide a course of his within the scope of practice of the nurse
or her own destiny? ● In the Philippines, to be sued for negligence it has to have
these four criteria, if one these are absent negligence
a. Beneficence cannot be sued:
b. Non maleficence ○ Duty on the part of the nurse
c. Autonomy ○ Failure to do the said duty
d. Justice ○ Lead to injury, harm, and death
The nurse is caring for a client who just gave birth to a healthy baby ○ Causation (the cause of the injury is the failure of
boy. The nurse may not disclose confidential information when: performing the duty)

a. The nurse discusses the condition of the client in a clinical Res ipsa loquitur- let the things speak for itself
conference with other nurses ● The injury caused by the failure of performance of duty
b. The client asks the nurse to discuss the her condition with will act as the evidence
the family
c. The father of a woman who just delivered a baby is on
the phone to find out the sex of the baby Which of the following situations would possibly cause a nurse to be
d. A researcher from an institutionally approved research sued due to negligence?
study reviews the medical record of a patient
a. Nurse gave a client wrong medication and an hour
A mother who is pregnant and has ovarian cancer has to undergo later, client complained of dyspnea
surgery to treat her cancer. In the process, the fetus died. The b. While preparing a medication, the nurse notices that
justification of the fetus' death is based on what ethical principle? instead of 1 tablet, she put two tablets into the client's
medicine cup
a. Justice
c. As the nurse was about to administer medication, the client
b. Beneficence
questioned why the medication is still given when in fact
c. Autonomy
the physician discontinued it
d. Double effect
d. Nurse administering 2 tablets of analgesics instead of 1
A client in the ICU executes a document that states all drugs he tablet as prescribed. Patient noticed error and complained
chooses to refuse in case his condition worsens. This is an example
Malpractice
of?

a. Living will
b. Power of attorney
c. Last will and testament
d. Informed consent

Martin signs a document authorizing his wife to transact any


document related to his illness condition while in confinement. This
document is?

a. Living will ● The red line represents the scope of practice of nursing
b. Power of attorney ● A nurse performing procedures that are beyond the scope
c. Medical records of nursing practice
d. Informed consent ○ Malpractice is still applicable even if there is no
harm, injury, or death so long as there is
You are one of the vaccinators in the Measles- Rubella Campaign.
performance beyond the scope of practice
One mother hesitated since she verbalized that it would hurt her son.
As a well-versed nurse about ethical considerations, you explained to
the mother that it might cause discomfort, but it will protect her son Nurses are already allowed to perform episiorrhaphy, they only
from acquiring the said diseases. What principle did you utilize? need to have a training and specialization

a. Nonmaleficence Nurses can do IE so long as there is no antenatal bleeding and if


b. Justice there are no fetal and maternal complications
c. Beneficence
d. Autonomy
Rape
Client Edwin was diagnosed with terminal cancer. His family asked
his physician to explain to him his illness on their behalf. What ● Follow the SRR rule
principle guides this situation?
The first nursing action when a person who appears frightened,
a. Autonomy disturbed and abused is brought in the ER?
b. Veracity a. Call the psychiatrist
c. Beneficence b. Put the patient at ease
d. The principle of double effect c. Take the vital signs
d. Get the chief complaints
Criminal Liabilities Affecting Nurses
False Imprisonment or Illegal Detention
Professional negligence
● Limit the patient’s movement from one place to another
● Except in these three cases:
○ (+) court order Simulation of Birth
○ Danger to himself or others
○ Extremely communicable diseases ● Tampering of the civil registry record to make it appear
in the record of birth that a child was born to a person
Albert's brother is deciding whether to commit him in a mental who is not such child's biological mother, causing the
institution or not. Albert refuses to cooperate. Albert can be advised loss of the true identity and status of the child
that his brother can only be treated involuntarily in a mental institution ● Switching of babies at birth, substitution of one child to
if? another child
a. Nobody can take care of him at home ● Destroying of the child’s identity
b. He has suicidal tendencies
c. Dangerous to others Assault
d. All of these
● The imminent threat of harmful offensive bodily contact
Who among the following patients cannot validly be committed in an ● Threatening the patient
institution even without permission?
Battery
a. Suspected polio inflicted patient
● Unlawful touching of another person without consent
b. Suicidal patient with mental disorder
● Unconsented touch
c. A psychiatric patient who disturbs a unit milieu therapy
Slander
d. Tourist, suspected of carrying illegal drugs
● Oral defamation
Which of the following patient's rights can be suspended for a
mentally ill patient? Libel
a. Right to privacy ● Defamation by written words
b. Right to treatment with least restriction
c. Right to freedom from physical restraint and seclusion Laws Affecting Nursing Practice
d. Right to confidentiality of records
History
Part of standards of care has to do with the use of restraints. Which
● Act 2808- first nursing law (1919)
of the following statements is not truc?
○ Before this act, the practice of nursing is under the
a. Doctor's order for restraints should be signed within 24 care of doctors
hours ○ After this act, it was placed under the board of
b. Remove and re apply restraints every hours nurses (members are initially only three)
c. Check the client's pulse. blood pressure and ● 1920- the first official board exam for nurses in the
circulation every 4 hours Philippines
d. Offer food and toileting every 2 hours
RA 9173 (October 21, 2002)
Putting restraint is a dependent nursing action except during Qualifications
emergencies. This should be reported to the doctor within 24
hours Board of Nursing

When a patient is placed on restraints, VS checking should be ● 10 - Ten years nursing practicum, but the last five years
done every 30 minutes. must be in the Philippines
● M - M.A.N.
The maximum hours for restraints are 2 hours, check ABCs ● A.- member of Accredited nursing organization
● T - Three year term in the BON
● S - Seven man team (1 chairman , six members, all nurses)
Abortion ● I - Immediately resign upon appointment to avoid conflict of
interest
● Termination of the product of conception before the age of ● N- Not convicted of any crime
viability (between 3-6 months or 12-24th week age of ● C- Citizen and resident of the Philippines
gestation)
Powers and Functions of the BON
Infanticide
● L- Licensure examination
● Killing an infant less than 3 days or 72 hours of life ● I- Issue certificate of registration after the oath
● M- Monitor the standards of nursing practice
Parricide
● E- Education, monitor nursing education
● Killing of a person to whom you are related to by blood ● C- Code of ethics, formulated
● H- Hear and decide cases (negligence & malpractice)
Homicide ○ Penalty: Suspension of license not more than four
● Killing of a person but is not premeditated years or revocation of license
● Without intent of killing ● A- Accredits different nursing organization
● Product of negligence ● G- guidelines for safe nursing practice in the Philippines

Murder Dean

● Killing of a person with premeditation ● RN, MAN with five years experience in the nursing practice
● With intent to kill Clinical Instructor
● A- accredited nursing organization A professional license of a RN is?
● M- MAN/other allied health courses
● O- one year clinical nursing practice a. Transferable
● R- R.N. b. Lifetime
c. Can be revoked by reasons stipulated under RA 9173
Nursing Service Administrators d. Personal

Nursing Licensure Examinees


Supervisor/ Community/ Military Hospital
Manager (head Government ● All these should be passed to the PRC before taking the
nurses) board exam
● CGM (optional)
● B- BSN/ RN
● Proof of valid holder of Filipino citizenship
● A- Accredited
● Proofs of valid holder of a BSN degree only from schools
organization
(PNA) whose curriculum is approved by the CHED
● N- Nine units How to be an RN under RA 9173
in nursing
management 1. –
● T- Two years 2. Take the exam
experience in 3. Acquire the required ratings
the general ● GWA of 75%, with ratings not less than 60% in any of the
nursing
practices
practice
● If there is a grade less than 60% in one of the subjects the
Chief/director add only master’s Same + general person is considered conditional, and s/he has to retake
RN + MAN + 5 om PHN or CHN staff course training the NP and the grade should already be 75% and higher
years supervisory
RA 8981: PRC modernization act
experience
(excluded if primary ● Announcement of board results are already done online
hospital)
● Primary hospitals- <50 bed capacity POST TEST
● For nurses who are assigned as chief nurses./ directors in 1. Patient C.G. has brain cancer with a poor prognosis. She has been
primary hospitals the qualification is similar to the admitted to the hospital for over two months now. Her condition is
supervisor/ manager in tertiary hospital deteriorating. She already meets the brain-death criteria. Which
nursing intervention is the most appropriate at this time?
Which among the following is the required qualification of the Dean in a. Make the decision to withdraw life support.
a Nursing school? b. Sedate the client.
c. Approach the patient’s family for possible organ
a. Nine units in nursing administration
donation.
b. Master's degree in related field d. Talk to the staff about their feelings
c. Master's degree in nursing
d. Doctorate degree
The most appropriate nursing intervention is to discuss with the
The faculty of a College of Nursing must have a Master's Degree in? family possible organ donation since brain-death criteria is already
met. The decision to withdraw life support is not within the scope
a. Psychology of nursing practice. There is no need for sedation because the
b. Nursing only patient is brain dead. Although talking to staff members about their
c. Public health feelings is also necessary, it isn’t the first action to take.
d. Nursing and related field

Nurses in nursing service must be qualified. The nursing service 2. Nurse Warren works with a colleague who consistently fails to use
director must have? standard precautions or wears gloves when caring for patients. Nurse
Warren calls the attention of the colleague but the colleague insists
a. Master's degree in any field that standard precautions and the use of gloves are not necessary
b. Master's degree in health science until the patient is known to have tested positive for HIV. What is the
c. Master's degree in nursing most appropriate action Nurse Warren would take?
d. Master's degree in public health a. Instruct the patients to remind this colleague to wear
gloves.
To be qualified, the nurse supervisor in the hospital must have at b. Document the problem in writing for the manager.
least? c. Discuss the issue with other staff members.
d. Ignore the problem; Nurse Warren has done his job in
a. Master's in Public Health reminding his colleague.
b. Nine units in master's degree in nursing
c. Nine units in nursing management at a graduate level Only a person has the right to discipline another nurse is a person
d. Master's of Arts in nursing higher in the hierarchy.
Nurse Warren has spoken to his colleague about the issue
Negligence in the practice of nursing can be a ground for:
properly but the nurse’s behavior hasn’t changed. Therefore, the
a. Revocation of license by the Ombudsman next step is to address the issue to the manager. It is inappropriate
b. Revocation of license by the BON to talk to other staff members about the issue because they don’t
c. Revocation of license by the DOH have the authority to bring the colleague’s practice to compliance.
A nurse should never point out to a patient that a staff member is
d. Revocation of license by the nursing department
not meeting the standards.
3. Nurse Jenny accidentally administers two capsules of phenytoin
Report unusual incidents to people higher than you in the
(Dilantin) instead of one capsule. The patient exhibits no adverse
hierarchy.
reactions to that larger dose. What is the appropriate action Nurse
Libelous statements in prints and pictures in the workplace are
Jenny should take?
ethically and legally prohibited. This should be addressed directly
a. Call the hospital’s lawyer.
to the nursing supervisor. Calling security is unnecessary. Option
b. Do nothing because the patient’s condition is stable.
B and D are not the most appropriate actions to take.
c. Document that she has given a capsule of phenytoin
(Dilantin).
d. Accomplish a complete incident report 7. Dr. Bailey ordered a bronchoscopy for Patient R.S. who has a
chronic cough. Nurse Camille brings the informed consent to Patient
R.S. for his signature. She then asks Nurse Camille to explain why
Always document all actions done, even if the patient is stable.
informed consent is necessary. Nurse Camille responds properly
Fully discuss what happened.
when she tells him that:
Nurse Jenny should file an incident report. Calling the hospital’s
A. The patient agrees to the procedure ordered by the
lawyer is not the job of Nurse Jenny. Taking no action is not an
physician, even if he does not understand what the
appropriate action. Documenting that she has given one capsule
outcome will be.
instead of two violates the virtue of veracity and the principle of
B. The physician must give the patient and his significant
non-maleficence.
others enough information to make health-care
judgments consistent with their values and goals.
4. Nurse Paulo suspects that Nurse Gilbert has been drinking alcohol C. The family of the patient will make the decision against the
before their night shift. He smells alcohol on Gilbert’s breath and patient’s will.
notes slurred speech. What is the best course of action Nurse Paulo D. The patient releases the physician from all responsibility for
should take? the procedure
a. Immediately notify the nursing supervisor.
b. Cover up for Nurse Gilbert because the profession
It is only the physician to explain the pertinent information about
depends on loyalty from colleagues.
the informed consent.
c. Tell Nurse Gilbert that he has one last chance, but if he
An informed consent requires the physician to give enough
drinks before another shift, he will be reported.
information to the patient and his significant others in order to
d. Call the security and have Nurse Gilbert arrested because
make decisions properly regarding the patient’s health. The patient
he endangers the lives of the patients.
needs to understand fully the procedure that will be done. It is
based on the patient’s wishes, not the family’s wishes, and the
Only a person higher than you has the right to reprimand a informed consent is not concerned about the physician’s
person. competence
A nurse who suspects another nurse of impaired practice has a
duty to report the colleague to the nursing supervisor. If Nurse
8. Nurse Suzette works in a prenatal clinic. She was informed by her
Paulo fails to do that, he may also face a disciplinary action. A
patient who is pregnant that her live-in partner (the father of the
nurse should not cover for an impaired nurse or give him one more
infant) has hit her in the past. The patient also told her that he is
chance. This action places patients in danger and prevents the
short-tempered and that she is worried about what might happen if
impaired nurse from receiving help. The nurse should report to the
the new infant has colic and he gets upset. She begs Nurse Suzette
nurse supervisor but not to the security or the police
not to report the problem because she is afraid that he will beat her.
Nurse Suzette should:
5. During her coffee break, Nurse Abby overheard a group of a. Record the incident in the nurse’s notes but do not report
residents passing by that a celebrity patient with syphilis is admitted for she did not actually witness the beating and she should
at the hospital last night. Having heard this, Nurse Abby went to her keep a promise to gain the trust of the patient.
friend Nurse Kate who was an avid fan of the celebrity. What ethical b. Record the incident in the nurse’s notes but do not report
principle was violated by Nurse Abby? for she will place the mother and the infant at greater risk of
a. Justice harm.
b. Confidentiality c. Record the incident in the nurse’s notes and report it
c. Informed Consent as reasonable suspicion of child and woman abuse.
d. Beneficence d. Divert the mother’s attention and teach her to breastfeed
her baby to reduce the possibility of colic
Never disclose personal information of any patient.
Medical information about a patient can be disclosed only to the A reasonable suspicion of abuse is created by the statements of
following; the patient, whoever is in-charge of the patient, those the mother. Healthcare providers are mandated by law to report
who may be affected by the patient’s health and to legitimate suspected abuse. The nurse needs not to witness the beating in
authorities or those who are directly involved in the care of the actuality. Diverting the attention of the mother is incorrect because
patient. it fails to deal with the legal obligation to report reasonable
suspicion of an abuse
6. Nurse Flor is waiting for the admission orders from the admissions
Follow the SRR rule:
department. After a few minutes, the runner of the admissions
● Safety of the client, if no safety is in the choices look for
department handed the order inside a folder. When Nurse Flor
words that implies safety (stay with the patient, provide
opened the folder, she was surprised to see a caricature of their
emotional support, silence).
hated hospital director with derogatory statements about her inserted
● Recording (record all the assessment findings as this
together with the orders. What is her most appropriate nursing
may be used as future evidence in court.)
action?
○ Burns
a. Call the nursing supervisor and report the incident.
○ Bruises
b. Call the runner back and ask for his name and the name of
○ Lacerated hymen, blood in the underwear
the person who sent the admission orders.
○ The child is complaining about the father
c. Call the security and have the runner arrested.
○ Never discard any clothe, article in the body of
d. Shred the photograph and keep it to herself for fun.
the patient. Keep it safe in a ziplock. (for DNA A, C and D are but part of the principle of confidentiality
testing)
● Reporting
12. Patient R.Q. who was rushed to the emergency room two hours
○ Under the VAWC law, report the incident
ago is now declared brain dead. Nurse Raffy has been with the
whether the parents of the patient want to or
patient since admission. Now, he must talk to the family about organ
not within 24-42 hours (to the barangay)
donation. Which of the following should be done by Nurse Raffy?
● Refer to services (DSWD)
a. Take the family to the chapel.
○ To keep the patient safe if the perpetrator is
b. Discuss life support systems.
within the home.
c. When discussing organ donation, include as many family
members as possible.
9. Nurse Ryan is assigned to provide postoperative care for Patient d. Clarify the family’s understanding and perception of
F.L. who has diabetes mellitus. During the assessment interview, the brain death.
patient reports that he is impotent and is much concerned about its
effects on his marriage. In planning for his care, the most appropriate
nursing intervention would be:
The family needs to understand what brain death is before talking about
a. Encourage the patient to ask questions about personal organ donation. They need time to accept the death of their family
sexuality. member. An environment conducive to discussing an emotional issue is
b. Provide time for privacy, the patient needs to examine his needed. Chapel is not the proper place where they would discuss their
thoughts. concerns. The phrase “life support system” may give family false hope
c. Provide support for the spouse or significant other. that the patient is still alive. And the more the family members are present
d. Suggest referral to a sex counselor or appropriate during discussion, the more difficult it will be to discuss issues and attain
professional unity in a decision.

Anxiety should be decreased by means of allowing the patient to


talk about feelings. 13. Nurse Giselle is caring for a woman who is pregnant and is
Nurse Ryan should refer the patient to a sex counselor or to other terminally ill with inoperable cancer. The woman has not been
professional. Making appropriate referrals is a valid part of informed about her status by the physician. The woman states to
planning the patient’s care. The nurse doesn’t normally provide Nurse Giselle: “I know I am terribly sick. I think you know what it is. I
sex counseling. want you to tell me right now.” Which of the following is the
appropriate thing for Nurse Giselle to do?
a. Give a detailed explanation of the disease of the patient.
10. Nurse Lorraine is caring for Mang Tiburcio, a homeless patient The patient needs to know.
who has active tuberculosis. The patient is almost ready for b. Deny knowledge about the patient’s condition and do not
discharge. However, Nurse Lorraine is concerned about Mang give the patient any further information.
Tiburcio’s ability to follow the medical treatment regimen. Which c. Make the patient’s chart available to her because after all,
intervention will best ensure that the client complies with treatment? she is entitled to see it.
a. Provide highly individualized patient education. d. Discuss the situation with the physician
b. Refer Mang Tiburcio to a social worker for discharge
planning.
c. Having the client attend a formal education session.
d. Carefully look for a member of Mang Tiburcio’s family It is the role and obligation of the physician to inform the patient. The
and ask for assistance nurse is obligated to inform the physician that the patient is demanding
information. The nurse has an ethical obligation to be truthful, but the
physician has the obligation to inform the patient. Regardless of
Involve the family in the care for continuous medical treatment. whether the patient has the right to view her record, the patient is not in
Referring the patient to a health care professional with knowledge the position to interpret the information found in the chart. Again, that is
of community resources is the best intervention to ensure the role of the physician.
compliance of a homeless patient. Educating the patient about his
health may help, but the basic necessities like shelter, food and
clothing must be met first. Providing formal education and 14. Leslie is a 27-year-old patient with lower gastrointestinal tract
attempting to contact family members are inappropriate when bleeding. She is a Jehovah’s Witness. The physician ordered 2 units
caring for a homeless patient. of packed RBC to be administered over 2 hours each. Upon knowing
this, Leslie strongly refuses and tells Nurse Kim, she won’t have it.
11. Patient B.D.L. is admitted to the medical unit with a diagnosis of Which of the following is the most appropriate response of Nurse
hepatitis B. Hospitals are mandated by accrediting bodies to Kim?
demonstrate respect for the patient’s confidentiality. Which of the a. Ma’am the physician’s order overrides your decision. We
following is a common violation of patient confidentiality? have to do this.
a. Keeping the medical records in the view and care of b. Ma’am, are you crazy? You’ll die if you won’t have this
healthcare providers directly involved with the patient. c. Kim approaches her supervisor and wishes to change
b. Placing a sign on the patient’s door stating “Isolation for patient assignment
Hepatitis B” for others’ safety. d. Ma’am, I understand your concern, I’ll inform your
c. Nurse making sure that the patient’s chart is secured at the physician about this.
rack in the nurses’ station
d. Providing the patient a copy of hospital policies and The patient has the right to refuse treatment. Inform the physician
procedures that address patient confidentiality at each unit. in cases like this because the physician will be explaining the
risks.
If the nurse is not involved in the patient care, they should not
view the patient's chart 15. In an emergency case, wherein the physician consents if the
The disease of a patient need not be in public view like in door patient is unconscious, which of the following is the belief that is
labels of patient’s rooms. Plain label of reverse isolation or strict upheld which states that another person knows what is best for
isolation is enough to ensure safety of healthcare workers. Options another individual?
a. Autonomy 2. “I’ll give you time to decide.”
b. Freedom 3. “Your wife will die without this surgery.”
c. Paternalism 4. “You’ve waited so long for a donor.”
d. Concern
a. 1 and 3
16. Cynthia has been the patient assigned to Nurse Jeff for one week b. 1,2,3
now. She reveals to Nurse Jeff that she is planning to commit suicide c. 2,3,4
and asked him to keep this a secret. Which of the following d. 1,3,4
statements can guide you in making a decision in response to
Cynthia’s plea? 22. Eva was rushed to the hospital after being found lying
a. Information may be revealed with the permission of the unconscious on the beach. Evidence shows that she was raped.
patient. Treatment of rape victims is one of the most ethically challenged
b. It is improper to reveal this type of information because it because it involves sexuality and life. Eva may opt to undergo a
will destroy trust which is important in any nurse-patient procedure that could prevent conception but may also be
relationship abortifacient if fertilization has already occurred. Which of the
c. Confidentiality should never be violated at all times. following is violated by this?
d. Confidential information may be revealed in serious a. Beneficence
harm is deemed to happen b. Non-maleficence
c. Human Dignity
d. Veracity
Confidentiality is not kept if it will cause harm to self and others.

Another term for human dignity is inviolability of life, prevention of


17. Nurse Gian carefully assesses his 83-year-old patient. He gently
anything that will remove the life of a person– euthanasia, abortion
assesses the patient to prevent injuring or hurting the patient.
This practice follows which of the following principles?
a. Beneficence 23. Mrs. Patricia De Castro is brought to the operating room for her
b. Non-maleficence scheduled cesarean section because the baby is in a breech
c. Confidentiality position. Nurse Laura should remember that which of the following is
d. Double Effect the most important intervention before Mrs. De Castro receives
anesthesia?
a. Proper skin preparation of the operative area
If there is (+) harm, it is always non-maleficence.
b. Insertion of the indwelling catheter
c. Obtain informed consent from Mrs. De Castro
18. Nurse Mabelle works in a health center. She gives a patient a d. Start and intravenous (IV) line
placebo drug. Which of the following principles is being violated by
Mabelle? 24. Ashley, 10 years old, is scheduled for a tonsillectomy at 7 in the
a. Double effect morning. Nurse Gelo is the operating room nurse who will bring
b. Autonomy Ashley into the operating room. Ashley asks Gelo if she can pray first
c. Veracity before they leave the room. Which of the following is the appropriate
d. Confidentiality response of Nurse Gelo?
a. “I’m sorry Ashley, we might be late for your surgery. We are
19. Mary Jane is a 20-year-old patient who is scheduled for strictly following the time.”
exploratory laparotomy tomorrow at 9 in the morning. During the b. “It’s alright. Will I stay here and pray with you or will I
morning of the surgery, which of the following should the nurse do? go out for a while?”
1. Assess the patient’s readiness for the surgery c. “Just pray by yourself on our way to the operating room.”
2. Give written and oral preoperative teachings d. “You should have prayed earlier. There are a lot of patients
3. Ask the patient’s relatives to sign the consent if the patient scheduled for operation today.”
is asleep.
4. Obtain and accomplish the preoperative checklist
Respect the beliefs and culture of the patient. But if it puts the
patient at risk you can prevent but explain why.
a. 1,2,3
b. 1,2,4
c. 1,3,4
d. All of the above
25. Proper disclosure of all needed information to the patient about
his condition and treatment alternatives is embedded in which of the
Paternalism does not apply to a patient sleeping.
following principles?
20. Nurse Minnie upholds the principle of social justice at all times. a. Double Effect
Which of the following precepts guide Nurse Minnie in doing this? b. Totality
a. Uphold universal access to healthcare and healthcare c. Autonomy
facilities d. Privacy
b. Collaborate efficiently with the members of the health team
c. Defend for the patient’s rights
Disclosing proper information, would guide the patient in proper
d. Do no harm to the patient.
decision-making

Whether the patient is rich or poor they should still be offered the
26. An unconscious 27-year-old male patient is rushed to the
same healthcare benefits.
emergency room after a vehicular accident. He has a closed head
injury. Which of the following legal documents will most likely be used
21. Nurse Paul knows that well-informed consent does not include to make a medical decision for the patient?
coercion. Which of the following statements depicts coercion or a. Special Power of Attorney
manipulation? b. Holographic Will
1. “Your family needs you with them.” c. Advance Directive
d. Durable Power of Attorney 2. Informed consent should exclude an explanation of
alternative treatments and procedures
27. Nurse Rupert knows that informed consent is important because 3. When a patient refuses to give consent, the husband or
it protects both the patient and the hospital from potential lawsuits. any significant other can sign.
Which of the following are exceptions to informed consent? 4. The doctor will decide what is best for the patient
1. Compulsory treatment is mandated by a court order
2. If patient relinquishes the right to consent a. 1,2,3
3. Emergency cases b. 1,2,4
4. The patient is an emancipated minor c. 2,3,4
d. None of the above
a. 1 and 3
b. 1,2,3 33. Before being hired as staff nurses, qualified applicants are
c. 2,3,4 subjected to mandatory HIV testing. This practice of hospitals shows
d. 1,3,4 which of the following ethical principles?
a. Beneficence
28. Which of the following patients are capable of giving an informed b. Non-maleficence
consent c. Totality
1. Married emancipated minor d. Double Effect
2. Blind adult
3. A manic-depressive patient 34. Nurse Lea has been asked to witness informed consent for
4. 25-year-old mute patient surgery. Nurse Lea understands that what she is witnessing is that
the
a. 1,2,3 a. Informed consent took place.
b. 1,2,4 b. The patient was fully informed about the procedure
c. 1 only c. Patient signs the consent form
d. None of the above. d. The family consented to the procedure

29. Which of the following situations exhibits the application of the


It is the legal obligation of the witness to verify that the signature
principle of justice?
took place. It is the legal obligation of the attending physician, not
1. Asking the poor people to become subjects in research for
the nurse, to conduct the informed consent process. It is the
drugs
attending physician’s obligation, not the nurse, to fully inform the
2. Providing quality health care equally to all patients
patient. And it is not necessary to obtain family consent
3. Delivery of the minimum health care to all according to
individual needs
4. Properly disclosing information regarding the procedure 35. Nurse Mylene works on the postpartum unit and is giving
and maintaining the anonymity of research subjects on medications at 9:00 am. She enters the room of a patient, calls the
abortion patient by name, checks the name band for accuracy, and
administers an oral vitamin. After the patient swallows the pill, she
a. 1,2,3 states that another nurse has also given her this same pill an hour
b. 1,2,4 ago. Mylene checks the medication record and finds that another
c. 2,3,4 nurse has signed off on the drug entry. The most appropriate action
d. All of the above. by the nurse is to:
a. Do nothing, because the patient was not harmed by the
30. Nurse Kristine receives an order from Dr. Velasco to give her second vitamin.
patient, Sammy Cruz two tablets of Ciprofloxacin (Ciprobay) for b. Report the first nurse to the nurse manager for not
urinary tract infection. Nurse Kristine knows that Sammy has no informing the medication nurse that a drug had been given.
urinary tract infection and that the drug should be given to another c. Submit an incident report describing the
patient who has a urinary tract infection. Which of the following circumstances of the medication error.
should Nurse Kristine do? d. Notify the patient’s physician and ask for advice.
a. Give the drug because it is the order of the doctor
b. Give the drug to the patient with a urinary tract infection
It is the legal and ethical obligation of the nurse to report incidents
c. Call the attending physician and clarify the order.
such as medication errors that exposes the patient to harm. Failing
d. Write a report and give it to the hospital lawyer.
to check the medication record to see if a drug had been given is a
form of medication error. Reporting the other nurse fails to
In case of errors or doubts in the orders, always clarify with the discharge the legal and ethical obligation to report any error.
physician. Actual harm to the client is not a prerequisite for filing an incident
report, and failure to report the incident does nothing to prevent
future incidents. It is not the legal and ethical obligation of the
31. Dr. Paredes ordered medication to be given intramuscularly on
physician to inform the nurse about the need to report a
the left deltoid of Mrs. Toledo. Nurse Percy knows that Mrs. Nolasco
medication error.
is post-left MRM. Which of the following should Nurse Percy do?
a. Give the medication on the right deltoid.
b. Give the medication on the right deltoid and document it on 36. Nurse Francis works in a hospice. Which of the following is a
the patient’s chart. characteristic of hospice care?
c. Follow the order. It is not contraindicated. a. Most of the patients have 6 months or less to live.
d. Inform Dr. Paredes that Mrs. Toledo is post right MRM b. Care ceases when the patient dies.
and clarify the order. c. Licensed personnel provide all of the care and comfort
d. Service is available at specified times
32. Ms. Tanjuatco is scheduled for proctosigmoidoscopy. She needs
to sign an informed consent before the procedure. Which of the
Hospice care is for patients who are terminally ill and their families
following is true about informed consent?
or caregivers. Bereavement counseling is available even after the
1. The patient has no ability to refuse the procedure after the
death of a patient. Educated volunteers are also part of the
consent is signed.
a. “Sure, you can tell me. Our code of ethics prevents me
interdisciplinary team. The service is available at all times. There
from telling anyone.”
is usually an RN on call 24 hours a day.
b. “Why not. You can tell me anything, you know.”
c. “I am responsible for the health of other people as well. I’m
37. Jason Ballesteros, RN, MBA is a nurse manager in St. Isidore sorry but I can’t keep anyone’s secrets.”
Medical Center. Caring for a patient with non-medical problems, he d. “I will not tell anyone who does not need to know. But
calls for the service of a social worker to address some of these if it is important to your health or someone else’s
problems of some patients. Which of the following patients in Jason’s health, I may have to tell another person. “
list requires assistance first?
a. Gertrudes Salvacion, 61 years old, post-stroke, lives alone.
The nurse is upholding the ethical obligation to tell the truth and to
b. Danilo Masangkay, 54 years old, with a history of recent
make the patient aware that, ethically and legally, the nurse may
myocardial infarction (MI), in a midst of a divorce
be required by law to report the information. That is not an
c. Artemio Romero, 72 years old, with heart failure, no
accurate statement about the code of ethics. Health of other
insurance coverage.
people does not necessarily force the disclosure of confidential
d. Mayumi Yamsuan, 29 years old, homeless, and newly
information.
diagnosed with diabetes mellitus

41. A nurse is morally opposed to abortion at any time during


Although all of the patients need social service and discharge
pregnancy. Which statement best describes the nurse’s responsibility
assistance, the patient who is newly diagnosed with diabetes is at
related to this belief?
greater risk of complications from the condition and the potential
a. The nurse must make this position known before being
for poor management. Proper diet and safe medication
employed at an agency that provides abortions.
administration would be difficult for a person who is homeless. The
b. The nurse may decline to participate in abortions but must
other patients are not faced with the most immediate risk.
care for women after the procedure.
c. The nurse cannot accept employment in any agency that
38. Nurse Celine is reviewing the medications in the physician’s order may provide abortion services.
and notes a medication with a dose that is two times greater than the d. The nurse must provide the same care for women
usual dose. Which of the following is the most appropriate nursing undergoing abortion procedures as for any other woman.
action of Nurse Celine?
a. Understand that there may be special circumstances in
Healthcare professionals need to identify and come to terms with
which additional medication is needed and give the
their own feelings regarding interruption of pregnancy. Nurses
medication as ordered.
whose religious or moral beliefs do not support abortion have the
b. Discuss the question about the drug with the physician
right to refuse the assignment of caring for patients undergoing
who ordered it and records the results of the
this procedure. The nurse should inform the employer of the
discussion.
beliefs before being assigned patients. The conflicts and doubts of
c. Check with the nurses from the previous shift to see if the
the nurse can be readily communicated to women who might
medication was given as ordered.
already be anxious and sensitive about the procedure. Nurses
d. Ask the patient if he or she has been receiving the dose of
have the right to refuse the assignment of caring for women
medication as ordered
undergoing abortion if their religious or moral beliefs do not
support abortion. The nurse may be employed in an agency that
The nurse is ethically and legally obliged to question orders that performs abortions, but may work in another section or unit.
appear incorrect in order to protect the patient from harm. The
nurse should not assume that the order is correct. The patient is
not an appropriate source to validate a medication order. The
actions of the nurses on previous shifts do not remove the
obligation to question that order, unless there is a clear note that
42. Shortly arriving for the evening shift, the triage nurse evaluates
the order has been questioned.
several patients who have come into the emergency department.
Which of the following patients should receive the highest priority?
39. Nurse Gabriel received his patient assignments for the shift. a. Jeff, a basketball player, limping and complaining of pain
Which of the following patients is Gabriel's priority in an acute care and swelling in his right ankle.
situation? b. Martin, a middle-aged man, is diaphoretic and
a. Delia who is newly admitted with chest pain complaining of severe chest pain radiating to his jaw.
b. Hope, who is complaining of pain following surgery for hip c. Celeste, with her 5-year-old son, who has been
pinning. complaining of nausea and has vomited once since noon.
c. Lulu with diabetes who has a glucose reading of 180. d. Pacita, an elderly who has been complaining of a loss of
d. Mary, who is three (3) days postoperative with left calf pain. appetite and fatigue for the past week.

The patient with chest pain may be having a myocardial infarction These are likely signs of an acute myocardial infarction (MI), which
(MI), and immediate assessment and intervention are priorities. is a cardiovascular emergency requiring immediate attention.
Although pain management is important, pain is an expected Acute MI is potentially fatal if not treated immediately. Pain and
manifestation from the surgery. Hyperglycemia of 180 is not life swelling is a musculoskeletal problem and not life threatening. A
threatening. Assessment and intervention is necessary but not child who vomited once since the start of complaint does not take
priorities compared with a patient with chest pain. Further priority over a life-threatening cardiac emergency. Loss of appetite
assessment of the patient with calf pain is necessary to rule out a and fatigue is not an immediate emergency,
thrombophlebitis and the possibility of a future pulmonary
embolism
43. Nurse Olivia joins a team who will respond in a calamity area. To
prevent mass hysteria during a disaster, Nurse Olivia would care first
40. Nurse William cares for a patient with Hepatitis B. The patient for which of the following patients.
calls him and says “Come here. There is something I want to tell you a. Manolo who is a drug addict
but you have to keep it a secret.” The most appropriate reply of b. Yolly who is depressed.
William would be: c. Perry who is euphoric
d. Sisa who is panicking nurse and no other bedside nurse. Which of the following should
Nurse April do?
a. Bring the infants out to their mothers to take care of them,
Patients who are in panic need immediate assistance because
this is mandated by the law anyway.
their behavior may result in harm to themselves or others.
b. Provide quality nursing care only to those patients regularly
Identifying addiction is more time consuming and not an easily
assigned to her.
solved problem. Depression would develop over a longer period of
c. Coordinate with the charge nurse and prioritize patient
time. Euphoria is defined as an exaggerated feeling of well-being
care so that all patients receive reasonably safe quality
(mild elation). It is unlikely that a patient who is euphoric would
care.
take priority over a patient who is panicking.
d. Refuse the nursing assignment because the increased
number of patients makes it unsafe not just to the patients
44. Nurse Nedy cares for a patient who wishes to donate his liver to but to the nurse as well.
his brother. What should be included in Nedy’s response when the
patient asks for factual information about becoming an organ donor?
In case of unusual circumstances, a test of reasonableness is
a. A donor can be of any age.
applied. The nurse is expected to be flexible enough to prioritize
b. Verbal consent would be enough.
care, and the minimal level of acceptable care is patient safety.
c. Family members must also give their consent.
Providing customary or ideal care to some patients and leaving
d. There are some contraindications for being an organ
others without care would not meet the test of reasonableness.
donor.
Refusing the assignment will expose the patients to harm from
abandonment. Care by family members does not obviate the need
The only factually correct answer would provide information about for nursing care or attendance by the nurse.
contraindications. Organ donors must be at least 18 years of age.
Written consent is required. Family members or legal guardians
48. Mr. Ventura is in the final stages of lung cancer. He tells Nurse
may authorize organ donation in the absence of written
Judy, “I wish I could just be allowed to die. I’m tired of fighting this
documentation by the donor.
illness. I’ve lived a good life. I continue my chemotherapy and
radiation treatment only because my family wants me to.” What is
45. It is important to consider the cultural and religious beliefs of the Nurse Judy’s best response?
patient in providing healthcare. What is the best reason to include a. “Would you like to talk to a psychologist about your
favorite or culturally required food in the diets of patients from thoughts and feelings?”
cultures other than the dominant one? b. “Would you like to talk to your minister about the
a. To prevent the patient from becoming agitated. significance of death?”
b. To prevent malnutrition c. “Would you like to meet with your family and your
c. To convey acceptance of the patient’s beliefs and physician about this matter?”
identity d. “I know you are tired of fighting this illness, but death will
d. To ensure the patient’s cooperation with scientifically based come in due time.”
treatment
The nurse has a moral and professional responsibility to advocate
Providing foods that are culturally acceptable to the extent for patients who experience decreased independence, loss of
possible is an important principle of cultural sensitivity when freedom of action and interference with their ability to make
planning nursing care. There is no guarantee that food from any autonomous choices. Coordinating a meeting between the
culture will decrease agitation. Specific foods are not mentioned. physician and family members may allow the patient an
Nutritional specifications are not included. In fact, favorite foods opportunity to express his or her wishes and promote awareness
may not be nutritionally sound. And even though serving culturally of his or her feelings, as well as influence future care decisions. All
required foods convey acceptance, it does not guarantee other options are inappropriate.
cooperation with other treatments.
49. One of the patients is dissatisfied with his hospitalization. He
46. Mr. Rajiv Singh is a 48-year-old Hindu with a severe necrotizing decides to leave against medical advice and refuses to sign the
ulcer of the lower left leg who refuses to undergo surgery because of paperwork. Which of the following is the nurse’s next course of
religious beliefs regarding reincarnation. The physician wants to action?
schedule a below-knee amputation of the left leg. What is the role of a. Detain him until he signs the paperwork.
the nurse in this situation? b. Detain him until his physician arrives.
a. Encourage the patient to have the surgery. c. Call the security for assistance
b. Discuss religious beliefs with the physician. d. Let him leave.
c. Call a family meeting.
d. Inform the patient of the other options available.
Ask for assistance from the security to prevent absconding.

The physician may not be aware of the role that religious beliefs
50. Mr. Clifford Jones, 92 years old with prostate cancer and multiple
play in making a decision about surgery. The nurse does not have
metastases, is in respiratory distress and is admitted to a medical
the right to encourage a specific treatment that is contrary to the
unit from a skilled nursing facility. His advance directive states that he
patient’s wishes. The patient has the right to make the decision.
doesn’t want to be placed on a ventilator or receive cardiopulmonary
The family could be incorporated when exploring all options to
resuscitation (CPR). Based on the patient’s advance directive, which
help the patient make an informed decision. And presenting other
of the following nursing interventions should the nursing care plan
options is the role of the physician. The role of the nurse is to
include?
assess whether the patient is well informed, relay information
a. Check Mr. Jones’ status once per shift.
pertinent to the decision and confirm that a decision is voluntary.
b. Provide mouth and skincare only if Mr. Jones’ family
requests it.
47. Nurse April is a staff member at the Newborn unit. On a particular c. Turn the patient only if he’s uncomfortable.
shift, two of the regularly assigned staff members are on sick leave d. Provide emotional support and pain relief.
and no new staff is available for assistance. She is with a charge
When advance directives state that a patient does not want life- without the patient’s consent is a breach of confidentiality. The
prolonging interventions, nursing care focuses on providing stigma associated with psychiatric illness may affect the patient’s
emotional and spiritual support and comfort measures. The patient employment: therefore, it is better to maintain confidentiality and
still needs to be checked regularly. The patient and the family refrain from disclosing
should not feel as if they have been abandoned. Providing mouth
and skin care makes the patient more comfortable. Turning the
patient provides comfort and prevents potentially painful 54. Ms. Lulu Yatco wishes to be discharged from the health care
complications such as pressure ulcers. facility against medical advice. What should the nurse do?

a. Call the security to help detain Ms. Yatco


51. Mrs. David became seriously ill after a nurse gave her the wrong b. Notify the physician of Ms. Yatco.
medication. When Mrs. David recovered, she filed a lawsuit. Who is c. Take measures to prevent the Ms. Yatco from leaving
most likely to be held liable? d. Request Ms. Yatco to sign a DAMA form.

1. The attending physician


If a patient requests for DAMA, the nurse should notify the
2. The hospital physician immediately. If the physician can not convince the
3. The nurse who administered the wrong medication patient to stay, the physician will ask the patient to sign a DAMA
4. The chaplain form. This form releases the hospital from legal responsibility. If
the physician isn’t available, the nurse should obtain the patient’s
signature on the DAMA form. A patient who refuses to sign the
a. 1,2,3 form should not be detained; forced detention violates the patient’s
b. 1 and 2 rights. After the patient leaves, the nurse should document the
incident thoroughly and notify the physician that the patient has
c. 2 and 3
left.
d. 1 and 3

Nurses are always responsible for their actions. The hospital is 55. Nurse Rachelle works in the pediatric unit of a tertiary hospital.
liable for negligent conduct of its employees within the scope of
As she goes back to the nurses’ station, she hears two family
employment. Consequently, both the nurse and the hospital are
liable. Although the mistake wasn’t intentional, standard procedure members arguing in a child’s room. They start to hit each other and
wasn’t followed. the child is crying. Which of the following should Nurse Rachelle do?

a. Call the security to come and intervene.


b. Remove the child from the room
52. A patient with end-stage liver cancer tells the nurse he doesn’t c. Ask one of the family members to leave the room
want extraordinary measures used to prolong his life. He asks what d. Try to reason with both family members.
he must do to make these wishes known and legally binding. How
should the nurse respond to the patient?
The first action would be to protect the child by removing her from
a. Discuss documenting his wishes in an advance the room. Calling the security is necessary but only after ensuring
directive. the safety of the child. Asking one of the family members to leave
the room or reasoning with them would be ineffective at this point
b. Tell the patient that it is a legal question and is beyond the
and may even escalate the situation
scope of nursing practice
c. Provide a copy of the patient’s bill of rights
d. Give information on active euthanasia
56. Nurse Jerome needed assistance in transferring Lola Carmela
who is confused, to bed. Nurse Jerome leaves Lola Carmela to find
Advance directives give a competent patient control over his someone to assist him with the transfer. While Nurse Jerome is gone,
situation and a legal forum in which to express his wishes about
Lola Carmela falls and hurts herself. Nurse Jerome is at fault
his care. Discussion of advance directives isn’t outside the scope
of nursing practice. The patient’s bill of rights involves multiple because he hasn’t:
patient rights and does not provide detailed information about a. Restrained Lola Carmela before leaving her alone
advance directives. Active euthanasia is illegal.
b. Properly instructed Lola Carmela about the safety
measures.
c. Arranged for continual care of Lola Carmela.
53. Max Perido, a 21 year-old call center agent, is admitted to the d. Properly documented that he has left Lola Carmel
psychiatric unit because of severe depression. Few days after his
admission, the employer of Max calls the nursing station inquiring
By leaving the patient, the nurse is at fault for abandonment. The
about the patient’s progress. The nurse doesn’t know if the patient
better course of action is to turn on the call bell or elicit help on the
has given consent to allow the staff to give information out to callers way to the patient’s room. Educating the patient about safety
on the phone. The best response of the nurse on the line is: measures doesn’t alleviate the nurse from responsibility for
ensuring patient safety. The nurse can’t restrain the patient without
a. “I’m sorry he is not here. I’ll just tell him to call you back.”
a physician’s order and restraints won’t ensure the patient’s safety.
b. “I’m sorry I can not confirm whether your employee is Documenting that he left the patient doesn’t excuse the nurse from
a patient here.” his responsibility for ensuring the patient’s safety
c. “I’m sorry sir. I am not in the position to discuss her
progress.”
d. “For a while ma’am, his attending physician is Dr. Mallen 57. A nurse in an ambulatory care clinic is performing an admission
and his number is +639235932056.” assessment for a patient scheduled for a cataract removal with an
intraocular lens implant. Which of the following questions would be
The nurse’s release of information to the patient’s employer the least important for the nurse to ask on an initial assessment?
a. “Do you have frequent episodes of headache?” a. Double Effect
b. “Do you have any difficulty in breathing?” b. Non-maleficence
c. “Do you ever experience chest pain?” c. Totality
d. “Do you have a close relationship with your family?” d. Privacy

64. As the nurse face ethical dilemmas, the principle of beneficence


Respiratory, cardiovascular and neurological assessments which
should be remembered for it means:
are physiological assessments are the priority assessment.
a. Do no harm
b. Do good
58. Mrs. Tanya Manzano is diagnosed with cancer and is told that c. Be truthful
surgery followed by chemotherapy will be necessary. She tells Nurse d. Be discreet
Cathy, “I have read a lot about complementary therapies. Do you
think I should try it?” Cathy responds by making which most 65. Which of the following professionals can declare the death of a
appropriate statement? potential organ donor?

a. ``No, because it will interact with the chemotherapy.” a. Transplant Surgeon


b. “You need to ask your physician about it.” b. The family lawyer
c. “I would try anything that I could if I had cancer.” c. Attending Physician
d. “There are many different forms of complementary d. Medico-legal
therapies. Let’s talk about these therapies.”
66.Which of the following are the qualities of a fully mature and
responsible conscience?
Although CATs should be approved by the physician and they may
have interactions with the treatment plan, the nurse should be 1. Free
therapeutic in dealing with patients with life threatening conditions. 2. Certain
3. Callous
4. Clear
59. Nurse Amanda hears a patient calling for help. She hurries down
the corridor and enters the patient’s room where she found the
patient face down on the floor. Nurse Amanda performs a thorough a. 1,2,3
assessment and assists the patient back to bed. The nurse notifies b. 1,2,4
Dr. Olalia, the attending physician of the incident and completes an c. 1,3,4
incident report. Which of the following would Amanda document in d. All of the above
the incident report?
67. The nurse can augment his or her ethical practice by doing the
a. Patient F.R. slipped from his bed. following, except:
b. Patient J.G. woke up from sleep terror and accidentally
jumped from his bed. a. Join in ethics committee and their forums
c. Patient N.J. was found on the floor face down b. Elucidate their own sets of values in life
d. Patient M.L. moved three (3) inches from the last seen c. Be familiar with the Medical Code of Ethics
position and jumped five (5) inches from the bed surface. d. Considering the values in life of other members of the
health team
60. As Nurse Sheila accurately and legibly in the patient’s chart, she
upholds which of the following ethical principles? 68. The Holy Scriptures provide a basis for Human Dignity through
which of the following?
a. Beneficence
b. Double Effect a. Easter
c. Totality b. Pentecost
d. Veracity c. Salvation History
d. Annunciation
61. As Nurse Glenn takes beneficent actions when he performs oral
and skin care to the patient and provides emotional support to those 69. Which of the following are the conditions that should be present
anxious or downed patients, he shows an application of which of the for the application of the Principle of Totality?
following?
1. That such member, by its deterioration in function, may
a. Beneficence cause damage to the whole organism or at least pose a
b. Non-maleficence serious threat to it.
c. Totality 2. That it is proven that the patient might be exposed to grave
d. Double Effect medical risks
3. That there is no other way than taking the indicated action
62. It is emphasized that cautiousness during medication against it of obtaining the desired good result.
administration is important to avoid errors which result in adverse 4. That the damage being avoided to the whole be
reactions of the medication. Lack of cautiousness in drug proportional to that which is caused by the mutilation of the
administration violates which of the following ideas? part.
a. “I will be faithful to the patient’s care.”
b. “I will be honest in my patient care.”
c. “I will avoid doing good to the patient.” a. 1,2,3
d. “I will avoid anything harmful to the patient.” b. 1,2,4
c. 1,3,4
63. When Nurse Kenny reports to the proper authority that Dr. Ku is d. All of the above
practicing drug abuse, he is upholding which of the following?
70. If the surgery did not push through as stated in the consent, and said: “We white nurses have more superior intellect that you
which of the following should be done? Asian Nurses.” Which of the following is the most appropriate
response of Nurse Joshua?
a. Obtain another consent for another day
b. Put a line across the date and time of the signed consent of a. “Yes, I agree.” So that there would be no commotion.
the postponed surgery, right the new date and time and b. “Honestly, I think you are a pathetic racist. But if that’s what
affix your signature you think, there is nothing I can do. It’s your opinion.”
c. Report the surgeon to the hospital legal counsel c. “I believe that we Asian nurses are far better than you white
d. Use the same consent form. It lasts until the procedure is nurses. Accept the fact that your country needs us because
done. we are skilled and smart.”
d. “I really don’t think that intellectual capacity and
71. Mrs. Delgado refuses to take her medications and consume her performance is related to race or color.”
food. She feels that she has no more reason to live since her
husband died last year. This situation is a direct violation to which 77. Which of the following is the ultimate destiny of human beings?
principle?
a. Utopia
a. Totality b. Preservation of life at all cost
b. Stewardship and Creativity c. Be in union with the Creator
c. Privacy d. Attain the self-actualization stage according to Maslow
d. Informed Consent
78.Which of the following is/are violations against the Principle of
72. Nurse Samantha cares for a male celebrity who is admitted for Stewardship?
hemorrhoidectomy tomorrow at 8 am. When she was asked by the
staff nurses from the neighboring ward what the actor was in for and 1. Habitual induced abortion of sex workers
she refuses to spill out the information, Nurse Samantha is applying 2. Below knee amputation (BKA) of a gangrenous leg
which of the following ethical principles? 3. Unauthorized deforestation
4. Fishing with the use of TNT and explosives
a. Stewardship and Creativity
b. Informed Consent
c. Beneficence a. 1,2,3
d. Non-maleficence b. 1,2,4
73. Mr. Samonte was recently diagnosed to have abdominal aortic c. 1,3,4
aneurysm (AAA) and is recommended for admission and surgery. He d. All of the above
refuses to be admitted to a first-class hospital and opts to be 79. Nurse Kelly is caring for a comatose patient. One afternoon the
admitted in a tertiary hospital due to financial constraints. Which of patient coded and no relatives were around. The patient does not
the following principles should guide Nurse Henry in making a have any advance directive. Which of the following professionals will
response? decide on behalf?
a. Justice a. Patient’s Lawyer
b. Totality b. Nursing Supervisor
c. Autonomy c. Hospital Chaplain
d. Beneficence d. Attending Physician
74. Nurse Valerie is working in a surgical unit and is assigned to 80. Dr. Vera Cruz requested Nurse Camille to accompany him to
post-operative patients. One of the responsibilities given to her is to secure a written consent from her patient who is for pelvic laparotomy
collect post-operative secretions either from the surgical site or from tomorrow afternoon. Which of the following requirements should be
the drainage collection. If one of the patients refuses and Nurse present so that written consent can be considered valid and legal?
Valerie still continues with the procedure, she is liable with which of
the following? 1. The patient should not be under the influence of alcohol or
drugs
a. Battery 2. The consent is secured with duress and coercion.
b. Assault 3. The patient is in the legal age of 18.
c. Negligence 4. The patient has received preoperative medications.
d. Abandonment

75. Upon returning to the Nurses’ station, Nurse Billy notices that a
personnel from the dietary department is looking through the chart of a. 1,2,3
a patient with hepatitis B. Which of the following nursing actions b. 1,2,4
would be the most appropriate? c. 1 and 3
d. All of the above
a. Inform the supervisor about the incident.
b. Shout at the personnel telling him that he is being 81. Nurse Portia received a call from the operating room and from the
inappropriate surgeon that the hypophysectomy scheduled for the patient tomorrow
c. Approach the personnel and tell him that it is not at 10 am is moved the next day at the same time. Which of the
allowed for those who are not directly involved in the following should Nurse Portia do?
patient care to browse through the patient’s record.
a. Put a line across the date written in the consent and
d. Do nothing because the person is part of the
replace it with the new date.
interdisciplinary team caring for the patient.
b. Secure a new written consent from the patient.
76. Nurse Joshua is a newly hired staff nurse in the surgical wing of a c. Act as a patient advocate and air complaint against the
hospital in Connecticut. One of his American co-staff approached him doctor.
d. Do nothing because consent is good up to seventy-two (72) 88. Once Nelia was appointed as a new member of the Board of
hours. Nursing, she is automatically disqualified to do the following except:

82. Mrs. Vergel de Dios is three (3) days post modified radical a. Guest speaker in a nursing school which has a poor
mastectomy (MRM) of the right breast. Nurse Glen accurately and performance level
legibly documents his care given to Mrs. Vergel de Dios. Which of the b. Resource speaker in a review center who was implicated in
following is being observed by Nurse Glenn? the leakage issue
c. Dean in a nursing faculty who received the highest passing
a. Confidentiality percentage
b. Privacy d. CHED Commissioner
c. Justice
d. Veracity 89. Renato has just passed the licensure examination for nurses.
Upon necessary oath, he received his professional identification card
83. One of the patients assigned to Nurse Farah is a terminally ill bearing the following, except?
patient who experiences severe pain. Nurse Farah knows that if she
gives the patient analgesics, it may shorten the life of the patient. a. Date of examination
Which of the following principles apply to this situation? b. Date of registration
c. Date of issuance
a. Justice d. Date of expiration
b. Double Effect
c. Totality 90. Nurses who have been inactive for five consecutive years cannot
d. Autonomy immediately return to practice of her profession, except if she has?

84. Nurse Rebecca saw Dr. Ho sniffing marijuana in the on-call room a. 1 month didactic/ 3 months practicum
of physicians. Nurse Rebecca immediately reported this to the b. 1 month didactic/ 5 months practicum
nursing supervisor. This action of Nurse Rebecca shows application c. 3 months practicum/ 1 month didactic
of which of the following principles? d. Upon completion of general staff course

a. Beneficence 91. For the chief nurse at a military hospital, special priority shall be
b. Non-maleficence given for those who have?
c. Privacy
d. Double Effect a. MAN
b. General Staff Course
85. Practicing basic hand washing and aseptic technique in doing c. Master in Public Health Nursing
procedures to patients shows exhibits which of the following? d. 5 months Military Course Training and Emergency Tactics

a. Beneficence 92. The three major areas of nursing practice are as follows, except?
b. Confidentiality
c. Double Effect a. Nursing Education
d. Non-maleficence b. Nursing Service
c. Nursing Research
86. When Nurse Lulu arrived for the evening shift, her supervisor d. Community Health Nursing
informed her that she will be pulled from the ward and will be
assigned to the understaffed emergency room. Nurse Lulu has no 93. Which among the following choices is not duly required under RA
experience but the special area rotation back in the nursing school. 9l73?
Which of the following is the best action Nurse Lulu would do? a. To take a fourth examination after flunking for three takes
a. Politely refuse to relieve in the emergency room. b. Nursing enrollees must belong to the upper 40% of his
b. Make an incident report and give it to the Chief Nurse. batch
c. Report to the emergency room and identify the tasks c. CHED has the power to close low ranking nursing schools
she can safely perform. d. Requires an applicant to prove his Filipino citizenship
d. Report to the emergency room and perform the tasks that 94. Which among the following basic qualifications is not required for
she saw the nurses do back in her rotation in nursing Alvin to be admitted as a clinical Faculty at Savio College of Nursing?
school.
a. Master's Degree in Public Health Nursing
87. Ms. Matilde Coronado, a 91 year old maiden is rushed to the b. Master's Degree in Nursing
emergency room due to elbow fracture. As Nurse Robert assesses c. One year clinical experience
Ms. Coronado, he notes old and new bruises on her arms and back. d. Licensed IV therapy nurse
When asked how she acquired those, Ms. Coronado told Nurse
Robert that her nephew hits her whenever she wets her clothes due 95. Which among the following shows that nurse Alvin is maintaining
to incontinence. But then she cried and told her that she has no continual learning to maintain competence in practice?
family and nobody takes care of her other than her nephew. Which of
a. Attending continuing professional education
the following is the most appropriate response of Nurse Robert?
b. Attending and finishing his MAN
a. I’ll instruct you with simple ways on how to control your c. Membership with the Philippine Nurses Association
urinary urge so that you won’t wet your clothes. d. Apply as a clinical Instructor or participate in research
b. I’ll try to look for some other relatives so that you can leave
96. One of the following choices are among the required
your nephew.
qualifications of a chief nurse at San Miguel Primary Hospital?
c. I would like to talk to your relatives
d. I’ll ask a social worker a. Completion of MA
b. 5 years experience as a nurse
c. Two years experience general nursing service
administration
d. GSC completion

97. The present composition of the Board of Nursing is?

a. 1 chairman, two members


b. 1 chairman 6 members
c. 1 chairman 7 members
d. 1 chairman 4 members

98. The institution which has the right to close a nursing school who
failed to comply with the minimum academic requirement is?

a. Professional Regulations Commission


b. Commission on Higher Education
c. Board of Nursing
d. PNA

99. An adult patient has just returned to the unit from surgery. The
nurse transferred him to his bed but did not put up the side rails. The
client fell and was injured. What kind of liability does the nurse have?

a. None
b. Negligence
c. Intentional Negligence Neo- new
d. Assault and battery Plasia- cells
Neoplasia- new tumor cells
The DNA contains the genetic structure of the cells
Cellular Aberration, Neuro-
Musculoskeletal
● Study of the detection, management, and treatment of Benign Malignant
cancers
Well-differentiated (looks the Undifferentiated (looks different
● Males- 1. lung cancer, 2. prostate, 3. colorectal (LPC) same like the normal cells) from the other cells)
● Females- 1. lung cancer, 2. breast cancer, 3. colorectal
cancer (LBC) Slow growth and proliferation Faster growth and proliferation
I. Pathophysiology Does not invade other cells and Infiltrates/ infiltration (invasion
● Cancer cells form when an abnormal cell is transformed by organs of the nearby cells) and
metastasis (goes to distant
genetic mutation of the cellular DNA
cells)
Genes (RNA/ DNA- genetic structure of the cells) are inside the Encapsulated Not encapsulated causing
nucleus faster proliferation
Mutation is the alteration of the RNA of viruses
In histopathology, normal cells are pinkish (d/t eosin stain) but Less chances to recur Increased chances of
tumor cells recurrence

Secretes hormones Secrete substances that


II. Risk Factors causes cachexia (muscle
wasting, skin and bones) and
easy fatigability
Endogenous Exogenous
Can be treated with surgery Treatment is a combination of
Genetics Radiation (too much heat
surgery, chemotherapy, and
causes changes in the
radiation (tumors are lysed first
cells→ mutation)
then removed through surgery,
Family history Viruses this will also depend on the
stage of the cancer)
Radiation is done to kill the
RNA cells of cancer cells

Tenderness upon palpation Nontender upon palpation

Soft and rubbery Stony hard

Regular shape and borders Irregular shape and borders

III. Carcinogenesis

● Production of new cancer cells


Stage 1: Initiation VI. Screening Exams

● Risk factors like chemicals, radiation, and biologic agents ● Initial exams, diagnostic procedures
(organisms) ● Under the level of secondary prevention
● For early detection, prevention, and management
Stage 2: Promotion

● Repeated exposure Detection of risk factors are under the primary level of prevention.

Stage 3: Progression 1. Breast Cancer


● Breast self-examination (BSE) should start at 20
● Increased malignant behavior d/t changes in the DNA of years old, monthly
the cells ● Clinical breast exam (CBE) should start at 40 years
old
● Mammography- x-ray of the breast, should start at
IV. Risk Factors 40 years old and done annually
1. Age- the young and the old are both immunocompromised 2. Colon Cancer (all exams under this is started at 50 years
making them more at risk of cancers old)
2. Alcohol- contains chemical substances ● Barium Enema (Lower GI series-LGIS)- xray
3. Cancer-causing substances- asbestos, arsenic, radon, UV ○ Barium sulfate is white (dye)
rays from the sun (avoid sun exposure from 8 am onwards) ○ This is for easier visualization of the
4. Chronic inflammation of a body part colon
a. E.g., constant inflammation of the larynx→ ○ The curvatures seen in the x-ray are
laryngeal cancer the tumors present
5. Immunosuppression (e.g., AIDS) ● Colonoscopy- direct visualization and biopsy, done
every 10 years
AIDS stage 3- CD4/ CD8 cells <200 and (+) opportunistic ○ Inserted through the anus and the
infections colon is completely assessed
Lymphocytes- kills virus but cannot killa CoVid-19 and HIV ● Fluoroscopy- ultrasound of the abdomen
● Proctosigmoidoscopy- visualization of the rectum
6. Infectious Agents up to the sigmoid colon, done every 5 years
a. Human papillomavirus→ cervical cancer and ○ Procto- rectum
penile cancer (warts may contain the viral ○ Before scopy, make sure to clean the
load) bowel first
b. Epstein Barr virus- Hodgkin’s lymphoma (painless ○ Sigmoid colon is the storage of feces
lymphoma) 3. Prostate Cancer
c. HTLV- leukemia ● Digital rectal exam (DRE), done at 50 years old
d. H. pylori- peptic ulcer- gastric cancer ○ The prostata is in the upper part
e. Schistosomiasis- urinary bladder cancer ○ Benign prostatic hyperplasia- benign
f. Hepatitis B- liver cancer ○ Prostate cancer- malignant
g. Hepatitis C- can be obtained from blood transfusion ● Serum PSA (prostate specific antigen)
because there is no screening test for hepatitis C ○ Antigen- foreign body→ tumor
7. Obesity- breast cancer (d/t estrogen receptors/cells) marker
8. Radiation- ionizing, radon (gas), x-rays, gamma rays, ○ (+) in prostate cancer
sunlight 4. Cervical cancer
9. Cigarette smoking- lung cancer ● Pap smear
● First hand smoking- directly smokes, active ○ 3 years after the first coitus
smoker ● (+) Risk factors- 20 years old annually (for sex
● Second hand smoking- passive smoking workers)
● Third hand smoking- can be acquired from ● (-) Risk factors- 30 years old every 3 years
objects that belongs to smokers

V. Warning Sign Lung cancers do not have a screening procedure.


Biopsy is the confirmatory test for cancer
● C- changes in the bowel and bladder habits (e.g., Proctitis is the side effect of external radiation of prostate cancer
constipated for two weeks then diarrhea or hematuria) Serum- blood
○ Colorectal cancers and urinary bladder cancer
● A- a sore/ulcer that does not heal
○ > 4 weeks- skin cancer VII. Diagnostic Procedures
● U- unusual bleeding or discharge ● Tumor Markers
○ Bleeding from the nipple- breast cancer ○ BRCA- breast cancer can be detected in the blood
● T- thickening or lump in the skin of people who are at risk of breast cancer
● I- indigestion (dyspepsia) ○ PSA- prostate cancer
● O- Obvious mole and wart changes (enlarges) ○ Ca125- ovarian cancer
○ Malignant melanoma ○ hCG in males- testicular cancer
● N- nagging cough or hoarseness ● Mammography
● U- unexplained anemia ○ Nursing Care
○ Leukemia can be manifested as intractable anemia ○ Avoid powder, lotion, deodorant will cause
● S- sudden weight loss (cachexia) whitish images in the x-ray that can be
○ Weight loss >5 kg- PTB, AIDS, or cancer mistaken as cancer cells→ false positive
● Magnetic Resonance Imaging (MRI)
● CT Scan- done next to UTS
● Fluoroscopy- ultrasound of the abdomen that is the
confirmatory test for pancreatitis, appendicitis, and
gallbladder colic
● Ultrasound- first done, cheapest
● Endoscopy- allows biopsy (second confirmatory test next
to biopsy) ● HPV causes genetic mutation of the cervical cells
● Biopsy- first confirmatory test ● Hodgkin’s lymphoma is cancer that can be seen in the
neck
Tumor Staging

VIII. Management

A. Surgery

1. Diagnostic surgery- biopsy


● Excisional- removal of the entire lump/tumor/mass
● Incisional- removes only a part of the tumor
○ If the lump is too large, incisional
biopsy is done
● Needle biopsy
2. Prophylactic- preventive surgery
● Mastectomy- removal of the breast
● Colectomy
● Oophorectomy (ovaries)
3. Palliative- to relieve complication and symptoms
● Tumor size ● Done in terminal illness/stage
● Nodes- local infiltration 4. Reconstructive- done after curative surgery
● Metastasis ● E.g., mastopexy after mastectomy

Stages Breast Cancer Cervical Cancer Hodgkin’s B. Radiation


Lymphoma
● Use of heat to lyse the cancer cells
I 1-2 cm Only confined One-sided ● Gamma rays are initially used
within the cervix enlarged ● Alpha and beta particles (most effective in lysing tumor
lymph node cells)
● Mechanisms of action of majority of radiation and
II 3-5 cm or Proximal ⅓ Two or more chemotherapeutic drugs- disrupts the generic structure of
axillary lymph enlarged the cells
nodes (4x4 cms lymph nodes ○ Will also affect the normal cells (affects the rapidly
on both sides in one side in dividing cells)
front of the ● Types of radiation- internal and external radiation
neck

III 6-7 cm Distal ⅔, Enlarged External Internal


already entered lymph nodes
the vagina on both sides Teletherapy Brachytherapy (done for
of the neck cervical cancer)

IV Distant Will travel towards Metastasis Radiation beam from a Radium implant (also done in
metastasis the front and back Reed machine (done for 30 minutes) cervical cancer)
primarily travels (towards the Sternberg
to the bone, urinary bladder cells will travel Patients are placed in isolation Patient is placed on complete
lungs, brain and rectum) to the room when radiation therapy is bed rest for 24 hours
liver, spleen done
Hematuria may (hepatospleno
present in this megaly) Nursing care:
stage 1. Systemic side effects
If (-) Reed- ● Most common effects
Hematochezia is sternberg are nausea and
fresh blood in the cells→ non vomiting, fatigue, and
stools hodgkin’s bone marrow
lymphoma suppression (risk for
bleeding), alopecia,
stomatitis,
xerostomia)
● The most common site of breast cancer is on the left ● Infertility, alopecia,m
breast near the axillary lymph nodes (upper outer stomatitis, and
quadrant of the left breast) xerostomia are
temporary because
this therapy also
affect rapidly dividing private room at the end of
cells the hall
● Also applies to
Bone marrow creates the communicable diseases
RBCs, WBCs, and ● Visitors are allowed for
thrombocytes 30 minutes only
If (+) BMS, stem cell therapy
will be done d/t decreased in
these three cells C. Chemotherapy
● Anemia- ↓RBCs
● Leukopenia- ● Goal- eradication of tumor cells
↓WBCs 1. Cell cycle Specific
● Thrombocytopen ○ often given more than once
ia- ○ Divided doses
↓thrombocytes ○ Destroy cells in the S phase

2. STD protocol (body


Cycles of Mitosis
fluids precaution)
● G0- resting phase
● G1- growth of cells
Shield- lead apron worn by the
nurse ● S- DNA synthesis (formation, development)
● G2- increased growth of cells
Time- 5 minutes per ● M- division of cancer cells→ proliferate faster than
intervention, in total of 30 the normal cells
minutes for the whole shift
● You can only enter 6
times
● Visitors can only be
inside for 30 minutes

Distance- 6 feet
● Do not position self
on the foot part of the
patient because the
brachytherapy is
placed near there
● Position self on the
sides
3. (+) radiation
A. Antimetabolites
● Body fluids precaution
● Fluorouracil
(urine, sweat)
● PPE ● Methotrexate (also given in h. mole and ectopic
● Gloves (sweat has pregnancy)
radiation) B. Plant Alkaloids
● Do not allow patients taking this to consume acidic
4. Double flushing foods
● Vincristine
5. Before internal radiation, ● Paclitaxel
insert indwelling foley
catheter 2. Cell cycle Nonspecific
● Cleansing enema to
prevent bowel movement A. Alkylating
as ordered ● Chlorambucil
● Cyclophosphamide (SE: hemorrhagic cystitis)
6. Low ● Cisplatin
fiber/residue/roughage B. Nitrosoureas
diet (x) fruits and ● Can cross the blood brain barrier
vegetables because this ● Carmustine
are high fibers→ (+) ● Lomustine
valsalva maneuver will C. Antitumor Antibiotics
dislodge the radium ● Bleomycin
implants ● Doxorubicin- SE: cardiotoxicity
D. Hormonal Agents
7. Long forceps placed in
● Prednisone, Dexamethasone (steroids)
lead container
● Once it dislodges, ● Tamoxifen (drug of choice for breast cancer)
another radium implants ○ Estrogen antagonist (estrogen is the
will be placed and 24 food for cancer cells)
hours will restart
Nursing Management
8. Place the patient in a 1. Assess fluid and electrolytes/blood chemistry
● The most common side effect is nausea and ● Hand hygiene- best prevention for infection
vomiting (handwashing-20-30 sec, alcohol)
● Complication: dehydration, metabolic alkalosis ● Avoid fresh flowers, fruit, vegetables (have live
(suka), hypokalemia (vomiting, diaphoresis, organisms that lead to opportunistic infections)
diarrhea)
D. Bone Marrow Therapy
Tumor Lysis Syndrome
● Allogeneic- from other human donors
● Autologous- from the patient themselves
● Syngeneic- from twins (their organs are compatible)

Nursing Care

● Harvesting of the bone marrow (bone marrow aspiration)


○ Common site (adult)- sternum
○ Common site (children)- posterior iliac crest of the
pelvis
○ Harvesting is done when the cancer patient is
health, when there are no signs of infection or
any compromise
● Transfusion is done intravenously
○ Done when there are signs of infection

E. Biologic Response Modifier

● Monoclonal Antibodies
○ Tocilizumab
○ -mab
● Group of disorders that is d/t the imbalance electrolytes d/t ○ Antibodies are synthetically copies (done inside
destruction of the cancer cells laboratories)
○ Antibodies destroy foreign bodies (cancer cells,
virus)
● Cytokines
○ From the immune system
○ Cytokine storm can result to ARDS (autoimmunity)
● Vitamin A, C, E (antioxidants)
○ Oxidation- no oxygen inside the cells d/t
↓ventilation (oxidative stress)
● Gene therapy
○ Genes from a person that attacks cancer cells
● Potassium has a direct relationship with phosphates→ (shingeki no cancer)
hyperkalemia and hyperphosphatemia
● Hyponatremia and hypochloremia- potassium has an Nursing Diagnosis
indirect relationship with sodium
● Phosphate has an indirect relationship with calcium→ ● Risk for impaired skin integrity
hypocalcemia ○ Maintain tissue integrity
● Tested and detected through blood chemistry ○ Mucositis/ stomatitis- saline gargle (+) sodium
○ Erythema, desquamation- use water only, avoid
soap, alcohol, lotion (this can cause irritation)
● Chemotherapeutic drugs are given through IV
○ Alopecia- offer a hair wig
● Should wear gloves and mask when being given
because this are also carcinogenic ● Imbalanced nutrition less than body requirements
● SE: mucositis (formerly stomatitis), diarrhea ○ Diet- carbohydrate + protein (for tissues)
● High uric acid- allopurinol, increase fluids ○ Cancer cells are hypermetabolic, thus, weight loss
● High potassium- insulin, calcium gluconate, kayexalate, is unavoidable
sodium bicarbonate, increase fluids ○ Small frequent feedings
○ Monitor weight daily
● Acute pain
2. Prevent bleeding ○ Administer morphine sulfate IV RTC (round the
● D/t bone marrow suppression- thrombocytopenia clock) / ATC (around the clock)
(low platelets) ○ Assess the severity of pain
● Use electric razor when shaving to prevent ■ Severe pain- 7-10
bleeding ■ Compilation is neurogenic shock
● Avoid invasive procedures (IM injections)
● High fiber diet to prevent constipation and valsalva Malignant cancer cells only become painful at the late or
maneuver advanced stages because it has already infiltrates adjacent
● Direct compression of 5 to 10 minutes for IM cells→ acute pain
injections
3. Prevent infection
● High protein diet (meats, legumes, eggs) ● Activity intolerance
○ Cannot perform activities of daily living
○ Easy fatigability ○ Breasts should be at the same level
○ Assess for fatigue ○ If (+) breast cancer, breast tissue goes up
○ Provide bed rest (if manifests muscle weakness) ● S- Skin dimpling (peau d’ orange)
● T- Tumor (painless)- initial sign is a painless, palpable
IX. Potential for Complications mass
Infection ○ Pain only occurs at advanced stages (stages 3 and
4)
● Monitor WBC count (5000-10000)- differential count ○ BSE- upper outer quadrant of the left breast
(counts the individual WBCs) ○ Men can also develop breast cancer
○ Most abundant is the neutrophils (40-60%)- kills
bacteria Diagnostic Screening
○ Second is lymphocytes (20-40%)- kills viruses ● Breast self exam
■ Cannot kill HIV and COVID 19- causes ○ For reproductive and fertile- BSE done monthly
lymphopenia because these two one week after menstruation because breast may
viruses consume lymphocytes be tender before menstruation
■ Lymphocytosis will be expected in ○ For infertile/non-reproductive/hysterectomy- BSE
other viruses done same day monthly
○ Third is monocytes (4-8%)- chronic TB, ● CBE (clinical breast examination)
macrophages ● Mammography- avoid lotion, powder, deodorant
○ Least in number eosinophils and basophils ○ To avoid false positive results
(minimal)- allergies ○ Is a painful diagnostic exam
● Septic shock d/t septicemia ● Biopsy (FNAB- Fine Needle Aspiration Biopsy)
Bleeding ○ If no biopsy in choices, check for aspiration
○ Most confirmatory test for breast cancer
● Monitor the platelet count (50,000- 400,000)
● Check for the RBCs (4-6 m/ cu. mm, Hgb (12-16 mg/dl), Management
and Hct (F: 36-46%/37-47%, M: 40-50%) 1. Surgical
○ Checked to determine active bleeding ● Radical Mastectomy
○ If ↓ (+) internal and external bleeding ○ Removal of the breast tissues, axillary lymph
nodes, nippel areola complex, pectoralis major &
Hemoglobin are inside the RBCs→ function is to bind with minor
oxygen ○ Complication: lymphedema
○ All radical surgeries remove the lymph nodes
● Modified Radical Mastectomy
BREAST CANCER ○ Except pectoralis to maintain shape of the chest
● Lumpectomy
● Malignancy of the breast
○ Removal of the mass only
● Infiltrating ductal carcinoma (malignant)
● Simple/Total Mastectomy
○ Removal/resection of the breast tissue only
Acini or alveoli creates the milk in the breast (includes nipple and areola complex if tumor
Lactiferous duct- passage of cancer cells and this is where cancer affects it as well) but pectoralis muscles are kept
cells infiltrate intact
2. Chemotherapy
● DOC: Tamoxifen- inhibits estrogen
Risk Factors 3. Radiation therapy- External
● Done for 30 minutes
● Family history
● Age Nursing Management
● Late menopause (>45 years old)
● Early menarche (<10 years old) Acute pain
● Nulligravida- no pregnancy ● Post-op
● Nullipara- has been pregnancy but (+) history of abortion ● Elevate the arm above the level of the heart after radical
(no 20 wks pregnancy) mastectomy to prevent lymphedema
● Those who do not breastfeed ○ Place the affected arm on a pillow
● High fat diet ● Administer narcotic analgesic IV as ordered
● Radiation exposure ● If (+) drainages→ jackson-pratt drainage may be
● Oral contraceptive pills placed
○ Strongest risk factor for breast cancer and ○ Removes inflammatory fluids that causes pain
cancer of the endometrium ○ Should be deflated at first to create negative
○ Contains estrogen pressure and create suction pressure
○ If (+) breast cancer, change to progestin pills ○ If drainage reaches ⅓, it should be emptied
● Alcohol, smoking because it will not suction discharges if it
Signs and Symptoms reaches that level
● Arm precaution- avoid BP monitoring and venipuncture at
● B- bleeding of the nipples the affected site
● R- retraction (inverted needle) ● Faster infections of the affected arm if radical mastectomy
● E- Elevation is done because lymph nodes are removed
● A- Asymmetrical
CERVICAL CANCER Nursing Diagnosis of all Cancers

Risk Factor ● Acute Pain r/t tissue injury from tumor invasion or surgical
incision
● HPV- human papillomavirus ● Knowledge Deficit
○ Warts in the genitalia is the initial manifestation ● Impaired physical mobility
○ Attacks the body once there is immunosuppression ● Ineffective coping
● Early coitus ● Ineffective sexuality pattern (cervical cancer)
● Multiple sexual partners (promiscuos) ○ Painful coitus- dyspareunia
● Low socioeconomic factor
● Immunosuppressed
1. Acute pain
● Uncircumcised male partner
● Administer narcotic/opioids IV
○ Smegma can harbor HPV
○ Narcotics can cause respiratory depression
○ If uncircumcised, prepuce should be cleaned
when overdosed
thoroughly
○ Infusion pump for accuracy
Signs and Symptoms ○ If RR <12, report to the physician
● To prevent tolerance and addiction, promote meditation
● Painless vaginal bleeding- initial sign and relaxation
● Serosanguineous vaginal discharge (clear reddish ● TENS- transcutaneous electrical nerve stimulation
discharge) ● Alternate heat and cold application
● Post coital vaginal bleeding
○ Cancer cells cause wounds in the cervix, once 2. Internal Radiation Therapy
penetrated it will cause bleeding ● Complete bed rest for 24 hours
○ Second sign if (-) painless vaginal bleeding ● Complication: DVT (blood clots) and thrombophlebitis
● Menorrhagia (Excessive bleeding), menorrhagia (bleeding (vein is already inflamed; very painful) d/t immobility
in between periods) ○ May be manifested as edema and/ or homan’s
● Back, pelvic pain (in the symphysis pubis) sign (pain of the calf muscle– gastrocnemius,
upon dorsiflexion)
Diagnostic Tests ○ Compression stockings may be worn to
prevent DVT, worn before insertion of the
● Pap Smear- done when there is no menstruation to avoid radium rods ay don
unclear results
○ Painless diagnostic exam but there is discomfort 3. Fear
● Colposcopy ● Encourage verbalization
○ Colpo- cervix ● Show empathy
○ Scopy- direct visualization
○ Visual of the cul de sac of douglas (space between 4. Check VS
● BP is always checked when (+) bleeding
the cervix and the bladder)
● Normal BP- 90/60 to 120/80
● CT Scan
● Below this→ hypovolemic shock
● Cone biopsy- confirmatory ○ PR is increased but BP is low in the
compensatory stage of shock
If (+) hematuria and hematochezia, cancer is already in the
advanced stages.
BRONCHOGENIC/LUNG CANCER

Management

● Internal radiation/brachytherapy (24 hours in bed)


○ Complete bed rest, walking and sitting is not
allowed because this will cause dislodgement of
the radium implants
● Panhysterectomy/total hysterectomy
○ Removal of the all the parts of the uterus (fundus,
corpus, isthmus, and the cervix)
○ No menstruation but (+) ovulation
● TAHBSO- total abdominal hysterectomy with bilateral
salpingo-oophorectomy
○ No ovulation and menstruation
● Radical Hysterectomy
○ Removal of the uterus, cervix, pelvic lymph nodes
○ Complication: lymphedema of the lower
extremities, infection
● Subtotal Hysterectomy-removal of the uterus except the
cervix
● Chemotherapy
○ 5-FU (five fluorouracil)
○ Carboplatin ● Tumor is usually located in the upper part of the lungs
○ Cisplatin ● Adam’s apple is located in the trachea
○ SE: bone marrow suppression
Risk Factors ○ Affected side up
○ Chest Tube thoracostomy- to remove drainages
● Smoking and re expand the lungs
● Pesticides/ insecticides ● Pneumonectomy- affected side lying
● Family history ○ Normal side up to promote lung expansion
● Immunosuppression ○ No chest tube is placed in this type of surgery
● Air pollutants- asbestos, coal dust because there is no lung left on the operated side
Symptoms
Closed Chest Drainage (CTT)
● Nonproductive/ dry cough- initial sign
● Wheezes can be heard
○ Tumor is obstructing the bronchi
● Pleuritic chest pain- felt upon inspiration
○ If (+) pain even after expiration→ MI or
angina
○ Seen in all lung disorders

(+) productive cough of a chronic smoker→ chronic


bronchitis
Foreign body obstruction- stridor can be heart (upper airway tract)

● Indication: pneumothorax, hemothorax


● Difficulty breathing
○ Thoracentesis- pleural effusion
● Hemoptysis d/t injury inflicted by the cancer cells (blood-
● On the first 24 hours, bloody amounting 500-1,000 mL
tinged sputum)
● Second day, if still bloody, refer
Diagnostic Procedure ● Bottle 1: Drainage bottle
● Bottle 2: Water seal
● Chest x ray- initial procedure ○ In the Intermittent bubbling during exhalation
○ But should be done in coordination with other tests and water will decrease in level; during
because it may be confused with PTB inhalation, water goes up and goes down
● CT scan during exhalation- fluctuation/tidaling
● Bronchoscopy ○ (+) continuous bubbling- air leaks
○ Biopsy is done first before any -scopy test if (-) ○ (-) intermittent bubbling- check for obstruction
biopsy ● Bottle 3: Suction chamber
○ Continuous bubbling
Biopsy should always be done first in cancers, if (-) results in
biopsy -scopy may be done.
● Sputum exam (not confirmatory)- morning specimen
Nursing Management
Management
● Acute pain
● Chemotherapy ● Ineffective airway clearance (priority- ABC)
○ Cyclophosphamide- hemorrhagic cystitis 1. Vital signs- O2 sat (N: >95%)
○ Cisplatin- bone marrow suppression ○ Low O2 sat- hypoxia (low oxygen in the tissues)
○ Doxorubicin- cardiotoxicity (heart failure) 2. ABG:
■ Digoxin (oral)- first given for heart ○ PaO2- 80-100 (>80- hypoxemia- low O2 in the
failure blood)
● Radiation therapy 3. Upright position- upright, semi fowlers
○ External radiation ○ Place the position first on upright before checking
● Surgery the VS
4. Breathing technique
Lung Surgeries ○ Pursed lip breathing
5. Administer oxygen at 3-4 LPM
● Segmental Resection- removal of one segment
○ Low O2 therapy in COPD
○ There are 10 segments in the lungs
6. High carbohydrate, protein diet (high caloric intake)
● Wedge Resection- small piece of lung tissue is removed
7. WOF complications:
○ Lungs will compensate the remaining lungs tissue
● Superior Vena Cava Syndrome
will enlarge
○ The superior vena cava collects blood from the
● Lobectomy- one lobe is removed
upper body
○ Right lung- three lobes (cancers develop on the
○ Once the tumor compresses the SVC→ SVC
apex)
syndrome→ jugular vein distention, facial
○ Left lung- two lobes
edema, headache, erythema
● Pneumonectomy- removal of one lung
○ Mgmt: radiation therapy to damage the DNA of the
Post-operative cancer cells causing the tumor to shrink (usually
done before the surgery for easier removal during
● Unaffected side lying for all the remaining lung cancer operation)
surgeries ○ Upright position to allow blood to flow back to the
○ To promote lung expansion SVC
● Spinal cord compression d/t tumor compression Diagnostic Procedure
○ Paraplegia- paralysis of the lower extremities
○ Bowel and bladder incontinence or continence ● Barium Enema (Lower GI Series)
■ Either patient does not have control of ● CT scan
the anal and urethral sphincter or the ● MRI
patrient cannot urinate or void ● Colonoscopy
○ Mgmt: steroids to decrease the inflammation of the ● Proctosigmoidoscopy- visualization of the rectum and
spinal cord sigmoid colon
● SIADH ● Serum CEA- tumor marker for colon cancer
○ D/t ectopic production of ADH by the tumor cells Management
○ Oliguria
○ Edema ● Chemotherapy
○ Hypertension ○ 5FU
○ Mgmt: Demeclocycline IV ● Radiation Therapy to reduce the size of the tumor
● Surgery
COLORECTAL CANCER ○ Abdominoperineal resection (APR)
Risk Factors ■ incision on the lower abdomen
■ Has two incisions, suturing, and post
● High fat, low fiber diet (low fruits and vegetables) op site
● Processed foods (has preservatives- nitrates) ■ A colostomy site will be placed after the
● Grilled foods surgery because sigmoid colon and
● Smoking rectum is already removed and anus is
● Obesity sealed
● History of polyps ■ Distal/Sigmoid colostomy
● Family history ■ Output will be formed (stools in the left
● Inflammatory bowel diseases. colon are already solid)
○ Crohn's disease- regional enteritis ○ Hemicolectomy
■ Inflammation of the small intestines/ ■ Half of ascending colon to sigmoid
ileum (pain in the right lower quadrant) colon
■ Symptom: diarrhea and vomiting ■ Transverse colostomy
○ Ulcerative colitis ■ Semi-formed stools
■ Inflammation of the large intestine (pain ○ Total Colectomy
in the left lower quadrant) ■ Only a part of ascending colon is
■ Symptom: bloody diarrhea retained
● Previous history of breast, ovarian, uterine cancer ■ Ascending colostomy- semi formed
stools
A person should defecate at least once a day to prevent colon ■ Ileostomy- watery stools
cancer (not defecating causes absorption of toxins)
Ostomies should be emptied when ⅓ full.

Symptoms
Colostomy Irrigation
Right colon cancer Left colon cancer
● To establish regular bowel movement
Seen in between the ascending Usually seen on the sigmoid ● NSS, tap water (can be used because the colon is not
and transverse colon colon sterile)
● Height: 12-18 inches above the shoulder
Stools in the right are still liquid Stools on the left are already ○ Same with the height of enema
solid ● Depth: 2-3 inches/ 3-4 inches
● Stoma: should be red and moist
Palpable abdominal mass Cannot be appreciated ○ Abnormal- bluish and violet (improper perfusion of
because there are already the stoma)
formed stools in the left colon
● Abdominal cramps: clamp temporarily and stop but can be
Abdominal pain- initial Changes in bowel habits resumed once cramps subside
manifestation (only cancer with (alternating constipation and ● Abdominal pain: stop and pull the tube out of the abdomen
pain as initial manifestation) diarrhea)- initial manifestation
LARYNGEAL CANCER
Anemia Rectal bleeding (rule out ● Larynx is a passageway of air
bleeding internal hemorrhoids ● Glottis are inside the larynx (vocal cords).
first) ● The most common site of laryngeal cancer is the glottis
○ Supraglottis, glottis, subglottis
Weight loss (cachexia) Tenesmus- frequent and urgent
feeling to pass stool, even if Risk Factors
bowels are already empty
● Cigarette smoking
Pencil-/ ribbon-like stools ● Alcoholism
● Voice abuse
● Chronic laryngitis
● Vitamin B deficiency (found in meat, pork) ● Anorexia
● Abdominal enlargement (hepatosplenomegaly) seen in the
Signs and Symptoms stage 4 (reed-sternberg cells travel)
● Hoarseness for more than 2 weeks Diagnostic test
● Sore throat
● Foul breath odor ● Biopsy- reed sternberg cells, if (-) reed-sternberg
● Odynophagia- painful swallowing cells→ non hodgkin's lymphoma
● Dysphagia ● CT scan

Diagnostic Test Management

● Laryngoscopy (if (-) biopsy) ● Early- external radiation therapy


● CT scan ● Surgery (removal of the lymph nodes)
● MRI
Radiation therapy is always done in the early stages of cancers.
Management
ENDOMETRIAL/UTERINE CANCER
● Radiation therapy- external
● Surgery ● Cancer of the uterus because of estrogen
○ Supraglottic Laryngectomy still (+) voice because
Risk factors
the glottis is not touched
○ Hemilaryngectomy still has (+) voice, only one ● Oral contraceptive pills d/t estrogen- unopposed estrogen
glottis is removed (does not have progesterone)
○ Total laryngectomy (x) voice because the ● Old age (post menopausal)
whole larynx is removed→ aphonia ● Obesity
● Diabetes mellitus
The larynx is hard while the trachea is soft, if the whole ● Family history
larynx is removed during total laryngectomy the trachea
will collapse→ to prevent this the trachea will be closed and Symptoms
a tracheostomy will be done
● Postmenopausal bleeding
Complication: ● Menorrhagia
● Dislodgement of the trach tube/ laryngectomy tube ● Metrorrhagia
● Late sign- pelvic pain

Management Menopause means no menstruation after 45 years old for one


year,
● For aphonia an electronic laryngeal device will be used to
allow speaking
○ Placed on the neck Diagnostic Procedures
○ Esophageal speech (training with speech
therapist)- learned technique in which air is ● Dilatation and curettage to remove the cancer cells- the
swallowed (“injected”) into the esophagus and specimen taken during D&C will be used for biopsy
then allowed to escape through the pharynx ● UTS/ Transvaginal ultrasound
● In case of dislodgement of the trach/laryngectomy tube, the ● CT scan
first things nurse should do is to open the airway by
Management
inserting the forceps
● TAHBSO
Nursing Diagnosis
● Total/panhysterectomy- whole uterus is removed
● Ineffective airway clearance ● Subtotal hysterectomy- cervix is not removed
○ Assess the RR, O2 sat, ABG
OVARIAN CANCER
○ Upright position (orthopneic)
○ Administer oxygen Risk Factor
HODGKIN’S LYMPHOMA ● Anovulation (sterile/infertile)
● Nulliparity
● Malignancy of the lymphatic system
● Late menopause (>45 years old)
Risk Factor ○ Too much estrogen causes late menopause
● Early menarche (<10 years old)
● Epstein barr virus ● Talcum powder
● Common in the younger age (10-20 years old) ● Family history
● Immunosuppression
● PTB Symptoms
● Family history/ genetic predisposition
● Abdominal distention d/t rapid proliferation of the cells
Symptom inside the ovaries
● Constipation d/t compression of the colon
● Painless enlarged cervical lump/mass ● Urinary frequency d/t compression of the urinary bladder
● Weight loss
● Night sweats Diagnostic Tests
● Abdominal UTS ● Cushing’s Triad (complete opposite of shock)
● CT scan ○ Hypertension (systolic)
● Paracentesis- aspiration of ascites ■ Wide pulse pressure (N: 30-40)
○ Ascites may develop in ovarian cancer ○ Bradycardia
○ Bradypnea
Management ○ This is different from cushing’s disease
● Oophorectomy ● Severe headache after waking up even after well rested
● TAHBSO ● Projectile vomiting
● Seizure d/t increased ICP
BRAIN CANCER

● Glioma- most common; benign tumor of the brain and


spinal cord Diagnostic Test
○ Increased ICP because of the tumor→ ● CT scan (biopsy and -oscopy not done for brain cancer)
compression of the brainstem that contains ● MRI
the breathing centers (medulla oblongata)
○ Common cause of death: brainstem compression/ Management
herniation
● Surgery
○ Craniotomy- large incision on the skull to remove
tumor or blood, etc., inside the skull

● Tentorium- fold of the dura mater forming a partition


between the cerebrum and cerebellum
● Meningioma ○ Incision is through the hairline
● Supratentorial craniotomy is done when tumor is in the
frontal, temporal, parietal, occipital area
○ Post op position- semi fowlers
● Infratentorial- Incision through the occipital
○ Done if (+) Tumor, hemorrhage of the pons,
medulla, and midbrain
○ Post op position- supine flat on bed

● Chemotherapy is secondary to surgery in cancer


treatment
○ Nitrosoureas (the only IV chemotherapeutic drug
● Transsphenoidal- procedure that removes the pituitary that can cross BBB)
gland; bleeding is checked through the nose ● External radiation therapy

Risk Factor

● Family history GASTRIC CANCER


● Radiation Risk Factors
● Genetic predisposition
● Raw foods
Symptoms ● Family history
● Restlessness, changes in LOC ● H. pylori- causes peptic ulcer
● Lethargy ○ H. pylori is also carcinogenic that causes genetic
mutation
● Old age Two hemispheres

Symptoms A. Frontal lobe


● Memory
● Indigestion/ dyspepsia ● Motor (gross)
○ Tumor obstructs the food, slows the gastric ● Abstract thinking
emptying time ● Judgment
○ Normal gastric emptying time: 2-3 hours ● Affect (emotion)
● Epigastric fullness even if (x) eaten ● Broca’s area- motor speech (located at the left side
● Nausea and vomiting of the frontal lobe)
● Epigastric pain (late sign)

Diagnostic Broca’s Aphasia


● Inability to verbalize or respond by talking
● Gastroscopy ● Nursing care:
● Abdominal ultrasound ○ Communication white board will be used by the
● CT scan patient to communicate

Management ● Abnormality: Atherosclerosis- cause of stroke, lipid


collection along the walls of the arteries
● Radiation therapy ○ Amnesia
○ to decrease the size of the tumor before operation ○ Muscle weakness (pareses)
● Surgery: gastrectomy ○ Paralysis (plegia)
○ Billroth I- Pylorus and antrum will be removed and ○ Problems in the motor:
the duodenum will be anastomosed with the ■ Agraphia: inability to write
upper half of the stomach- corpus ■ Apraxia: inability to perform
○ Billroth II- pylorus and antrum will be removed and motor movements
the jejunum will be anastomosed with the upper B. Parietal Lobe- Sensory
half of the stomach- corpus ● Pain, pressure, texture, temperature
○ Duodenum is not removed because the pancreatic ● Abnormalities: Agnosia
duct, common bile duct (liver and gallbladder is ● Astereognosis- inability to recognize familiar
connected to the duodenum objects especially by touch
● Complication: dumping syndrome ○ Patients will be instructed to close their
eyes and recognize the object placed
on their hand.
● Paresthesia: numbness
C. Temporal Lobe
● Auditory
● Understanding of language and music
● Wernicke’s areas- seen in both the right and left
temporal lobes
● Abnormalities: Wernicke’s Aphasia

Broca’s Aphasia Wernicke’s Global Aphasia


Aphasia

Seen in the left Seen in the left and Both the broca’s
hemisphere of the right temporal lobe and wernicke’s area
frontal lobe; more are affected
Nursing Care common

● Position should be supine to prevent increased movement Inability to verbalize Cannot understand Cannot understand
and dumping of the hypertonic chyme or respond by spoken language and speak
● Diet- low carbohydrates, moderate protein, high fats talking
● Avoid fluids
Communication Communication Communication
board used by the board will be used board will be used
Zinc increases immunity. patient by the nurse by the patient and
the nurse

Neuro
D. Occipital Lobe
Anatomy and Physiology ● Vision
● Visual interpretation
Central Nervous System
● Abnormalities:
● Head and body ○ Blurring of vision (but more on the
● Brain and spinal cord cranial nerve VIII)
○ Dyslexia- difficulty reading from the
Brain right or the left (nonexistent movement
in words and seeing letters like “d”, “b”, ● Helps regulate respiration
“p”, “q” rotated) C. Medulla oblongata (middle)
○ Alexia- inability to read ● Respiratory center
● Cardiac center- regulates blood pressure and pulse
Thalamus rate
● Relay center of sensations ● Reflexes- cough, sneezing, swallowing /gag,
● Pain manifestations are taken to the thalamus→ vomiting
travel to the parietal lobe for interpretation ● Vasomotor- movement of the blood vessels
○ Tumors in the thalamus or parietal lobe will not be ○ Vasoconstriction- ↑BP and pulse
able to interpret pain rate
○ Vasodilation- ↓BP and pulse rate
Hypothalamus ○ Affectation of the medulla may cause
fluctuations of BP and pulse rate
● Thirst center
● Emotional response
Projectile vomiting is manifested with the affectation of the medulla
● Sleep center
oblongata.
● Temperature regulation
● Endocrine functions
● Sexual arousal
If the brain stem herniated and the pons varolii is compressed the
● Increased ICP (manas ang utak) decrease of LOC and patient will immediately die.
thirstiness will manifest when hypothalamus is affected
● Hypothermia Herniation or compression of the (1) pons and (2) medulla will
cause death.

Herniation in the uncus→ uncal herniation


Basal Ganglia

● Control of fine motor movements (writing)


● Substantia Nigra- black pigmented neurons
○ Dopamine
○ Affected in Parkinson’s disease
● Huntington’s chorea (clumsiness)
○ Problems in the caudate Nucleus and putamen will
result to this condition
● Cerebral cortex is the outermost layer of the brain

● Innermost part of the temporal lobe

Meninges

● Epidural space- space between the cranium and dura


mater
Brain stem (most important part of the CNS) ○ Does not have CSF
● Subdural- space between the dura mater and arachnoid
● Meninges- covering of the brain and the spinal cord
mater
○ Dura mater
● Subarachnoid- space between the arachnoid mater and pia
○ Arachnoid mater
mater
○ Pia mater
○ (+) CSF- cushions the brain (makes it float; if it
● Spinal cord is connected to the medulla oblongata
does not float it will compress the brain stem)
A. Midbrain
● Auditory Cerebrospinal Fluid
● Visual reflexes
● Sensory and motor pathways (fine motor) ● 500 ml/day
● Helps the frontal lobe in motor movements ● Clear
B. Pons varolii ○ If cloudy, it indicates infection
● Apneustic center- affectation of this part will result ● Specific Gravity- 1.007
to sudden cessation of breathing ● WBC: none or minimal (d/t leukocyte circulation)
● RBC: none (+) RBC may indicate hemorrhages or bleeding
● ↑Glucose
● ↓Protein

if ↓glucose is and ↑protein, there is bacterial meningitis


Viral meningitis: normal CSF
Cerebellum

● Balance/equilibrium
● Proprioception
○ Awareness of where each body part is
○ Awareness of the position of every body part
● Fine motor- helps dopamine Posterior/ Dorsal horns

Brain/Cerebrum ● Sensory
● Needs to receive 15% of cardiac output (4-6 LPM)
● First organ to be damaged when (+) blood losses SPINAL CORD INJURY SYMPTOMS

Characteristics ● Most common cause is vehicular accidents


● Arteries- carry blood away from the heart ● Paresthesia
● Veins- have valves and delivers blood back to the heart ● Paralysis
○ Veins of the brain do not have valves! ● Bowel (fecal) and bladder (urinary) incontinence
○ Arterioles (capillaries) of the brain has only two ○ Continence- does not excrete (e.g., constipation)
layers ○ Incontinence- cannot control excretion (e.g.,
■ Tunica adventitia urination cannot be controlled)
■ Tunica Intima
○ If PTT, PT, INR is too high (too much Emergency Management
anticoagulants)→ high turbulence in the
brain will cause rupture of the arteries and ● Immobilize cervical spinal cord (head and neck should
veins be in proper alignment)
○ The phrenic nerve is here (C3-C5)-- innervates
Blood Brain Barrier the diaphragm
● Log rolling (3 people- head, body, lower extremities)
● Protective function
● Mannitol can cross the BBB
● Can be altered by head trauma, cerebral edema,
hypoxemia Spinal Nerves
Spinal Cord
Spinal Nerves (pairs) Vertebrae (pairs)
● Housed by the vertebrae
● From the foramen magnum to L1 vertebra Cervical- 8 Cervical- 7

Anterior/ Ventral Nerve Roots (horns) Thoracic- 12 Thoracic- 12

Lumbar- 5 Lumbar- 5

Sacral- 5 Sacral- 5

Coccygeal- 1 Coccygeal- 1

Total: 31 Total: 30

Cranial Nerves

● 12 pairs (24 in total)


● I- Olfactory Nerve
○ Anosmia- partial or full loss of smell
● II- Optic
○ Blurring of vision
● III- Oculomotor
● Motor function that supplies the nerves below/ lower ○ Ptosis (may be seen in myasthenia gravis)
extremities ○ Anisocoria- one is constricted while the other is
● Abnormality: Paresthesia, paralysis dilated (cerebral edema, increased ICP)
○ Strabismus
● IV- Trochlear
○ Strabismus, ptosis, anisocoria
● V- Trigeminal- largest nerve
○ Trigeminal neuralgia (pain d/t viral infection)
■ Inflammation of the lower two nerves
○ Three branches in both sides of the face
↑BP ↑SBP (only, systolic
● VI- Abducens
hypertension; diastolic BP
○ Strabismus, ptosis, anisocoria remains the same)
● VII- Facial
○ Bell’s Palsy (paralysis of the facial nerve) ↑Pulse rate and respiratory Widening pulse pressure
■ Affects the organ in the ears that rate Bradycardia
dampens the sound→ autosensitivity
■ Photosensitivity Vital signs only manifest when the person is nearing death (late
○ Asymmetrical facial expression sign) in increased ICP, but all other disorders will manifest
● VIII- Auditory, Acoustic band, Vestibulocochlear changes in vitals signs as early signs.
○ Vestibule
■ Meniere's disease (vertigo)
■ Rupture of the reinesser;s membrane ● Decreased LOC→ deep coma
that separates the endolymph and
Diagnostic
perilymph
■ Tinnitus ● CT scan- cheapest diagnostic procedure first
■ Sensorineural hearing loss (deafness ○ Confirmatory diagnostic test for increased ICP
on one side) ● MRI- not for claustrophobic people
○ Cochlea ○ Assess for claustrophobia
■ Organ of corti (hearing) ○ Remove metals
■ Interprets what you hear
● IX- Glossopharyngeal Skull x-ray for skull fractures only
● X- Vagus- longest nerve

CN 9 and 10- dysphagia Management


● XI- Spinal accessory I. Mannitol (osmitrol)- osmotic diuretic
○ Inability to shrug both the shoulders ● DOC for cerebral edema
○ Sternocleidomastoid muscle injury ● Head injury will cause inflammation→ third space
● XII- Hypoglossal
shifting
○ Controls the tongue
● MOA: given through IV→ cross the BBB→ mannitol
○ Lateral deviation of the tongue→ dysarthria
will pull the plasma from the interstitial compartment
(difficulty in speaking; slurred speech)
to the intravascular compartment→ fluids will go to
DISORDERS kidneys for excretion
● Nursing interventions:
INCREASED ICP ○ Monitor the hourly urine output (30-60 ml/hour),
multiply by three because people urinate every
● Cerebral edema
three hours
● Death may be d/t brain stem herniation
● Side Effect: hypotension, dehydration, hypovolemia,
● Normal ICP: 0-10 mmHg
pulmonary edema
Causes ● Can also be given for acute glaucoma
● Head injury- most common cause d/t falls II. Dexamethasone- steroid
● Brain tumor III. Furosemide- potassium wasting diuretic
● Preeclampsia ● Furosemide will pull the plasma pulled by the mannitol that
● Hydrocephalus is left in the lungs
● Subarachnoid hemorrhage (severe headache) ○ Prevents pulmonary and cardiac edema that is
caused by mannitol
Pathophysiology ● SE: hypokalemia
Increased bulk of the brain (glioma- most common tumor)→ IV. Anti Seizures- diazepam (valium) or phenytoin (dilantin)
Cerebral ischemia (hypoxia)→inflammation and edema→ ● Convulsions can lead to further hypoxia of the brain
decreased brain tissue perfusion→ (1) shifting of the brain V. Antipyretic- acetaminophen (paracetamol)
tissues→ (2) brain stem herniation→ respiratory arrest VI. Anti-ulcer
● H2- ranitidine
Manifestations ● PPI- omeprazole

● Initial manifestation is restlessness, confusion, agitation,


Patients with increased ICP should be NPO.
changes in the level of consciousness
○ Restless is manifested because the most sensitive
part of the body is being agitated
Nursing Diagnosis
● Projectile vomiting
● Ineffective airway clearance
Do not allow the patient to sleep→ may go to coma ○ Airway first before breathing
● Anisocoria ○ Always used for bronchial asthma and COPD
● Blurred vision ● Ineffective breathing pattern
● Severe headache upon arising in the morning ○ Tachypnea, bradypnea
● Cushing’s triad- late sign of ICP ● Ineffective cerebral tissue perfusion

Interventions for all the diagnoses


1. Assess VS, LOC, GCS 7. Assist in Intubation (ET tube)
● Inserted by the anesthesiologist
● Connected first by ambu bag or mechanical ventilator
○ Hyperventilate to expel carbon dioxide→
decreased retention will cause respiratory
alkalosis (carbon dioxide is acidic, when it
combines with water it becomes H2CO3 or
carbonic acid)
○ Hyperventilation will cause cerebral
vasoconstriction (alkalosis will cause this) that
will decrease the ICP
○ Hypoventilation (slow, shallow breathing)→
retention of CO2 (respiratory acidosis)→
vasodilation→ increased ICP
○ Bradypnea only slow breathing
● ABG is done everyday for people who are connected to the
mechanical ventilator
8. Provide a quiet and non-stimulating environment
● Only enter the room when you need to do interventions

Surgical Management

● Supratentorial craniotomy
○ Frontal, parietal, temporal, occipital
● Pinch the shoulders to determine decortication and
○ Semi-fowler’s position postoperatively
decerebration
● Infratentorial craniotomy
● Decorticate- flexion of the arms
○ Brainstem, cerebellum
● Decerebration- extension of the arms
○ Supine/ flat on bed
● Coma- no response
● People get confused with time first
● Brain injury levels Craniectomy is done then is placed in the peritoneum to nourish it
and prevent decay if the patient is still undergoing craniotomy.
○ Mild- 13-15
Cranium is only placed back once edema has subsided
○ Moderate- 9-12
○ Severe- 3-8
○ Deep coma- 3
CEREBROVASCULAR ACCIDENT

Obsolete: ● Aka stroke, brain attack


15- normal ● Neurologic deficits d/t decreased blood flow to a localized
11-14- lethargy area
8-10- stuporous ● Most common cause of CVA is atherosclerosis
4-7- light coma (responsive- opens eyes)
● 7 is the start of coma Risk Factors
3- deep coma
● Completely unresponsive Nonmodifiable Modifiable

2. Semi fowler's position (30 °-45 °) with the head and neck Age Obesity
aligned
Gender Hypertension
● To promote venous drainage/ outflow to the heart and
● ≥140/90 mmHg for
prevent compression of jugular vein→ cerebral vein
both diastole and
distention d/t poor venous drainage→ increased ICP systole
3. Administer oxygen at 3-4 LPM
4. Administer mannitol and dexamethasone intravenously Race Diabetes Mellitus
5. Suction secretions PRN
● Duration should be less than 10 seconds only, suction Family history Atherosclerosis - most
upon withdrawal common cause
● Hyperoxygenate/preoxygenate the patient for 5 minutes to
prevent hypoxia Genetic predisposition Substance abuse (cocaine has
6. Avoid the following: the largest particles and can
clog the brain vessels)
● Valsalva maneuver (bearing down)
○ Administer laxative (Lactulose at HS- hora somni) Hyperlipidemia
● Prolonged suctioning
● Coughing, sneezing, blowing of nose Sedentary lifestyle
● Rectal proc- thermometer, enema, exam
● High flexion Transient Ischemic Attack- less
● Rationale: To prevent vagal stimulation that will cause than 24 hours, after some time
bradycardia the patient will experience
○ Patient is already bradycardic, prevent further stroke
decrease of heart rate
Ischemic Hemorrhagic

Thrombotic Embolic Hemorrhage

Most common Most fatal type of


type of stroke stroke

Older age (>60 Younger adults/ Chronic


years old) middle-aged adults hypertension
(20-30 years old) (arteries become
weekend lead to
aneurysms→
rupture)

Atherosclerosis of Rheumatic heart


the middle cerebral disease,
artery (this is where Myocardial
the cerebral artery infarction,
● Agnosia
bifurcates), or Atrial fibrillation (no
internal cerebral P wave and not ● Dysphagia
artery uniform), Congenital ● Slurred speech (hypoglossal)
Heart Defects, ● Dyslexia
Valve Disorders ● Agraphia- frontal lobe
(blood clots from ● Incontinence
the heart that can ● Aphasia- lack of communication
go to the brain) ● Unilateral neglect- inability to take care of the other side of
the body affected by paralysis or weakness

LADA (left anterior descending artery) is the most commonly Hemiparesis or hemiplegia are the first manifestations of CVA.
affected by atherosclerosis in myocardial infarction and angina
pectoris
Diagnostic Test

● CT scan- most common diagnostic procedure for CVA


Pathophysiology
● MRI
● ↓blood flow→↓cerebral tissue perfusion→↓glycogen, ● Cerebral angiography- confirmatory test for CVA
ATP, sodium-potassium pump failure→ cerebral
Management
edema→ penumbra (dead brain cells, dark area in the
brain) ● Mannitol
● Within 5 minutes without oxygen the brain will die ○ Osmotic pressure- pulls
○ For cerebral edema
Symptoms
● Dexamethasone
● Manifestations will depend on the artery affected ● Furosemide- to prevent pulmonary edema
● For cranial nerves 1-4, 5-8 will not decussate ● Anticoagulants
○ Manifestations are on the ipsilateral side ○ IV Heparin (check PTT)-
● Manifestations will appear on the contralateral side d/t ○ Oral warfarin (check PT)
○ Prevents blood clots in the veins (DVT)
decussation of pyramids (crossing of nerves at the medulla
○ Not for hemorrhagic stroke
oblongata at the level of the eyes)
○ Indication: ischemic stroke
○ CN 9-12- contralateral manifestations ○ AE: bleeding
1. Middle cerebral artery ● Antiplatelet aggregate
● Hemiparesis- one sided weakness ○ Dipyridamole, Clopidogrel, Ticlopidine, Aspirin
○ Affectation of the frontal lobe (acetylsalicylic acid; aspillet)
● Hemiplegia ○ Prevent blood clots in the arteries (arterial
● Homonymous Hemianopsia- Loss of half of vision thrombosis)
○ E.g., if there is right-sided CVA the patient cannot ○ To prevent MI (LADA), stroke (MCA)
see the right nasal and left peripheral ○ Taken after lunch
○ Optic nerve is affected ○ Not for hemorrhagic stroke
○ Indication: ischemic stroke
○ AE: bleeding
● Thrombolytics
○ Streptokinase; tPA, alteplase, teleplase,
tenecteplase
○ To dissolve blood clots
○ Given within 3 hours
■ >3 hours- permanent blood clot
● Calcium Channel Blockers
○ Lower blood pressure
● Antiseizure/Anticonvulsant
Nursing Management
● Ineffective cerebral tissue perfusion ○ Drugs- haloperidol (atypical, first generation
antipsychotic), reserpine, methyldopa
For MI always prioritize acute pain
Pathophysiology
1. Assess VS
2. Monitor SpO2 (N: >95%) - hypoxia ● Depigmentation of the substantia nigra (inside the basal
○ ABG paO2 (N: 80-100 mmHg) - hypoxemia ganglia) that creates dopamine→ ↓dopamine,
3. Semi-fowler’s position with the head and neck aligned to ↑acetylcholine→ skeletal muscle rigidity
prevent compression of the jugular vein that causes
cerebral dilation Parkinson’s disease is the only disorder that affects the substantia
4. Mannitol and dexamethasone (prevents inflammation) nigra
5. Suction secretions PRN (<10 seconds) DOC: Levodopa
● >10 seconds→ vagal stimulation→
bradycardia
6. Avoid the same interventions in increased ICP Symptoms (MTB)
7. Administer oxygen
● Tremors- unintentional, at rest, pill rolling
● Bradykinesia- slow movements
● Impaired physical mobility ● Stooped posture- leaning forward
1. Encourage range of motion exercises to prevent joint ● Shuffling, festinating gait (from slow to fast gait, hips are
contractures used to move forward)
2. Turn to sides q2 hours to prevent pressure ulcer ● Muscle rigidity
3. WOF DVT ○ Micrographia- small writing
4. Administer anticoagulants especially heparin ○ Mask-like face
○ Microphonia
● Cognitive defects
● Impaired swallowing ○ Dementia
1. Feed with SAP (strict aspiration precaution) ● Autonomic nervous system imbalances (constipation,
2. Upright position (high fowler’s- during eating) urinary retention, dysrhythmia)
● Semi fowler’s after eating ○ All d/t rigidity of the muscles
3. Put the food in the unaffected side with head slightly flexed ○ Bladder- detrusor muscle
○ Dysrhythmia- myocardium
Flex the head, to prevent aspiration, to close the epiglottis. ○ Intestines- smooth muscles

Hyperextension opens the epiglottis Diagnostic Exams

● PET scan- parkinson’s


○ Visualizes the lightened substantia nigra
● Impaired urinary elimination
● MRI
1. Bladder training q2-3 hours (only allow the patient to
urinate every 2-3 hours) Medical Management
2. Kegels exercise
3. Increase oral fluid intake 1. Dopaminergics
● Levodopa (Sinemet- combination of levodopa and
carbidopa)
● Impaired skin integrity ○ Carbidopa prevents destruction of the levodopa as
1. Turn to sides q2 hours it ascends towards the blood brain barrier
2. Chair bound- lift the hips q15 minutes ● MOA: stimulate dopamine secretion
3. Use a cushion
Nursing care

● Disturbed visual sensory perception ● SE:


1. Approach from the unaffected side of vision ○ Dark urine
2. Turn the head side-to-side ○ Dyskinesia
○ Orthostatic hypotension
PARKINSON’S DISEASE ○ Hallucinations
● Progressive degenerative ○ Arrhythmia
○ Progresses if not addressed
○ Degenerative- comes with age Admitted in the basement to prevent injuries, cannot climb stairs
● Avoid B6- inhibitory; does not absorb levodopa
Osteoarthritis- most common degenerative (comes with age) ○ Baboy (pork), meats
● Second most common degenerative disease to 2. Anticholinergics
alzheimer's disease ● Benztropine (Cogentin)
● Causes: ● MOA: inhibits the transmission of acetylcholine in the
○ Idiopathic- unknown cause muscles
○ Old age 3. MAOIs
○ Viral infection ● Antidepressants
○ Carbon monoxide poisoning damages the 4. Diphenhydramine (Benadryl)- to sedate the patient and
substantia nigra avoid muscle rigidity
Nursing Management
The thymus glands atrophies at 6-7 years old
Impaired physical mobility Thymoma- thymus gland >7 years old

● ROM
○ Active- the patient moves the extremity Thymoma→ autoimmune response→ (1) increase in
■ Moves the unaffected side cholinesterase (2) increase in antibodies→ destruction of the
○ Passive- the nurse moves the extremities myoneural junction that contains the acetylcholine receptors→
■ Moves the affected side (-) nerve impulses→ flaccid muscles (soft muscles)
● Frequent ambulation if tolerated
● Use assistive devices (walker) Manifestations

Risk for Complications ● Ptosis (MG starts from the eye manifestations)
○ Mata Galing
● Constipation- Increase Oral fluid intake, High fiber diet ● Diplopia (double vision)
● Stool softeners- laxatives (lactulose) ● Mask-like face (no facial expressions d/t flaccid muscles)
● Prevent joint contractures (ROM or prone position- allows ● Drooling of saliva (d/t to flaccid facial muscles)
straightening and extension of all joints in the body) ● Dysphagia
○ Prone positioning aligns all the joints ● Dysarthria
● Splinting may also be done ● Descending paralysis
● Respiratory paralysis d/t paralysis of the diaphragm
● Nerves innervate the muscles ○ Late sign of both MG and GBS
● Myelin sheaths- covering of nerves that allows passage
of acetylcholine Diagnostic Tests
○ Acetylcholine is excitatory ● CT scan of the chest to r/o thymoma
● All movements are d/t the work of acetylcholine ● Serum Ab
● Dopamine balances acetylcholine in the nerves that ● Electromyography- decreased action potential
prevents rigidity of muscles ● Tensilon test- IV edrophonium is given in incremental
● Acetylcholine is located in the myoneural junctions doses
● Acetylcholine receptors (cells) are found in the ○ Most confirmatory test for MG
myoneural junctions
● Acetylcholine needs to bind to these receptors to create
Myasthenic Crisis Cholinergic Crisis
impulses that is needed to fuel up the muscles
Undermedication Too much edrophonium
Pathophysiology of guillain-barre, myasthenia gravis, multiple
sclerosis, and Parkinson's disease will be derived from this. Respiratory paralysis are manifested for both, ptosis

Tensilon test: improvement of Tensilon test will further


Guillain- Myasthenia Multiple Parkinson’s muscle strength (positive weaken the muscles (negative
Barre Gravis Sclerosis Disease tensilon test) tensilon test)

Demyelination Destruction of Demyelination Depigmentati Management- Mestinon Management- Atropine


of lower the myoneural of central on the (cholinergic- push Ach→ (Anticholinergic)
extremities junction that nerves substantia excitatory)
(peripheral contains the nigra, no ● Tensilon test is given when (+) respiratory distress and
nerves) Ach receptors problem in the ptosis
myelin sheath
and the Management
myoneural
junctions ● Cholinergic- pyridostigmine (Mestinon)
● MOA: stimulates acetylcholine transmission/
Muscles will Flaccid Spastic Rigid muscles conduction to the myoneural junction→ increased
become soft paralysis paralysis (+) nerve impulses
(flaccid spasms,
paralysis) intentional Nursing Care (Mestinon)
tremors
● SE: (parasympathetic manifestations)
○ Bradycardia
MYASTHENIA GRAVIS ○ Diarrhea
○ Urinary frequency
● A neuromuscular disorder that results in failure to ● Administer before meals- AC (ante cibum)
transmit nerve impulses in the myoneural junction
● Damage in the myoneural junction
● Causes:
○ Idiopathic ● Plasmapheresis- filtration of plasma to remove the antigen-
○ Autoimmune antibody complexes
○ Thymoma (tumor of the thymus gland) ● Steroids- anti inflammatory
■ Thymus gland has not atrophied ● Immunosuppressants
■ Thymectomy is done to prevent MG ○ Azathioprine
○ Cyclosporine
○ Cyclophosphamide ○ The only acute disorder of the myelin sheath is the
GBS
Nursing Management
Causes
Ineffective airway clearance
● D/t diaphragm weakness ● Unknown
1. Assess for signs of hypoxia ● Viral infections
2. High fowler’s position ● Viral immunizations
3. Administer oxygen ● Autoimmune diseases
4. Prepare trach set at bedside (long term)
● 14-21 days Pathophysiology
5. Deep breathing exercises, coughing exercises q2hrs to ↑T cells in the CNS → inflammatory process→ destruction of
prevent pneumonia myelin and oligodendrocytes (creates myelin sheath of the
Impaired swallowing CNS)→ demyelination

1. Administer Mestinon 30 min ac Manifestations


2. Small frequent meals with small bites of foods (half spoons) ● Cerebrum
3. Adequate rest before meals ○ Fatigue
GUILLAIN-BARRÉ SYNDROME ○ Depression
○ Mood swings
● Acute inflammatory demyelination ● Brain stem
● Causes: ○ Nystagmus- eyes make repetitive, uncontrolled
○ Viral infection movements.
○ Bacterial infection (campylobacter jejuni) ○ Dysarthria
○ Viral immunizations ○ Deafness
○ Vomiting
Pathophysiology
○ Vertigo and tinnitus
B cells and T cells look for the antigen→ destruction of ● Optic nerve
myelin sheath of peripheral nerves→ poor conduction of nerve ○ Diplopia
impulses→ flaccid paralysis ○ Blurred vision
○ Scotomas- aura or blind spot that obstructs part of
Manifestations your vision
■ Dark spots around the vision
● Paralysis of the legs- initial manifestation
● Cerebellum
● Severe pain- when sensory nerves are unmyelinated
○ Ataxia/ ataxic gait- unsteady, staggering gait
○ Stocking glove pattern
○ Hypotonia
● Paresthesia
● Spinal cord
● Ascending paralysis
○ Weakness below
● Respiratory paralysis
○ Numbness
○ Trach set should always be at bedside
○ Lhermitte’s sign- electric shock-like sensation that
Diagnostic Test occurs on passive flexion of the neck

● CSF analysis- lumbar/ spinal tap Diagnostic Test


○ ↑proteins d/t inflammation
● Spinal tap- increased in IgG (most predominant antibody)
○ normal glucose
and T lymphocytes
● Electromyography
● MRI
○ Decreased action potential of the muscles
● CT scan
Management
Charcot’s Triad of multiple sclerosis (SIN)
● Intravenous Immunoglobulins that will suppress antigens
● Immunosuppressive drugs 1. Scanning speech
● Plasmapheresis- removes antigen-antibody complexes 2. Intentional tremors
● Anticoagulants d/t DVT 3. Nystagmus
● Morphine sulfate (narcotic analgesic) for severe pain
○ Codeine also (look for other opioids)
Management
Nursing Diagnosis
1. Steroids
Acute Pain 2. Plasmapheresis
1. Assess pain severity 3. Immunosuppressive drugs
2. Provide morphine sulfate IV as ordered 4. Baclofen- antispasmodic
3. Apply alternate cold and heat compress
● 10 minutes alternating cold and heat compresses
Nursing Management
Impaired Physical Mobility
Fatigue
MULTIPLE SCLEROSIS

● Chronic demyelinating disease of the CNS


● Prioritize activities in the morning (this is where energy is ○ Carbamazepine (tegretol)
high) ■ DOC: Partial/focal seizures
● Perform tasks in the morning ○ Phenytoin (dilantin)
● Avoid extreme temperatures and stress ■ Causes gingival hyperplasia→
gum bleeding
SEIZURES/ CONVULSIONS ■ Soft toothbrush
● Almost same as epilepsy ■ Massage the gums
● Causes ○ Gabapentin
○ High fever in pediatric patients (immature ○ Phenobarbital
hypothalamus in children) ■ Infant
○ Brain infections (meningitis, encephalitis) ○ Ethosuximide
○ Increased ICP ■ DOC: petit mal/ absence seizure
○ Alcoholic withdrawal- do not abruptly stop ○ Diazepam (valium)
○ Hypoxia ■ DOC: tonic-clonic seizures
○ Electrolyte imbalances ○ Lorazepam (Ativan)

Pathophysiology

Alterations in the excitability of neurons, imbalance in the


acetylcholine and gamma aminobutyric acid (GABA; Nursing Diagnosis
inhibitory neurotransmitter)→ (1) both hemispheres -
generalized; (2) one hemisphere - partial/focal 1. Risk for injury
○ Priority is safety in seizures
Generalized Seizure ● Raise padded side rails (if patient is in the hospital)
1. Tonic-clonic seizures/ grand mal ● Avoid restraints
● 20 seconds of rigidity, 20 seconds of nginig ● Community- place the patient on the floor
● Convulsions should only be for 30-60 seconds ○ Place pillows on the head to prevent skull fractures
○ If it does not stop, immediately bring to the hospital ● Side Lying position
because of status epilepticus ● Left side lying- for pregnant women to prevent compression
● Aura presents before a seizure- warning sign of the vena cava
● Apnea
● Chewing of tongue
● Loss of consciousness 2. Ineffective airway clearance
● Muscle rigidity d/t increased acetylcholine ● Loosen clothing
● Alternating muscle contraction and relaxation ● Turn to side
● Incontinence ○ To allow the tongue to go to side to prevent airway
● Highest priority obstruction
● Avoid putting a tongue depressor or spoon in the mouth
○ May cause blockage of the epiglottis
2. Petit Mal/ Absence (more common in children) ○ Causes teeth to break
● No loss on consciousness ● Administer oxygen via face mask
● Blank stare ● During seizure, observe the patient, time the seizures (<30
● Mouth twitching sec)
● Loss of motor activity ○ >1 min- irreversible (status epilepticus)
○ DOC: diazepam q15 min; SE: respiratory
depression d/t relaxation of the diaphragm
Partial/Focal Seizures
Musculoskeletal Disorders
1. Simple
● Aura Anatomy and Physiology
● Hallucination
Skeletal System
● No loss of consciousness
● Skeleton- group of bones
● Adult- mature bones >7-18 years old
2. Complex ● Children- cartilages (immature bones)
● Lip smacking or picking at clothes ○ The ribs are cartilages, thus, easily breakable
● Loss of consciousness ● Parts
○ Axial- 80
Diagnostic Tests ○ Appendicular- 126
● Electroencephalogram Functions
● Skull x ray- to check for fractures
● CT scan- to check for brain tumors ● Bone formation
● Blood glucose- hypoglycemia can lead to convulsions ○ Vitamin D, calcium (taken together to increase
● Lumbar puncture absorption)
○ Parathyroid hormone- makes bones sturdier
Management ● Articulation- joints
● Muscle contraction
● Anticonvulsants
● RBC production in the bone marrow
○ Flat bones in adults
○ Long bones in children (femur)
○ RBCs, WBCs, and platelets come from stem cells

Types of Bones

● Long bones- radius, ulna, tiba, fibula, femur (longest bone)


● Short bones- metacarpals
● Irregular- vertebrae
● Flat- sternum Diagnostic Test

● ESR and CRP (erythrocyte sedimentation rate and c-


● Middle part of the bone- diaphysis
reactive protein)
● End of the bone- epiphysis
○ Increased when (+) inflammation
○ Elongates
● X-ray of the joints
○ Stops at 18 years old in females
○ Joint space narrowing may be seen
○ Stops at 21 years old in males
● Arthrocentesis- aspiration of synovial fluids
Bone Cells ○ Cloudy (inflammation and infection)
○ WBC
● Osteoblasts- immature blood cells
○ Confirmatory test
● Osteoclasts- bone resorption
● Rh factor- (+) antigen
○ Increased in osteoporosis
○ C- cuts bones
● Osteocytes- mature
Management
Synovium
1. DOC- aspirin (asmalin)- nonsteroidal
● Has synovial fluid that prevents friction of bones ○ Salicylates
○ Has narrow therapeutic effects
RHEUMATOID ARTHRITIS
○ NSAIDs- naproxen
● Autoimmune, inflammatory disorder of the joints
Toxicity:
● Juvenile rheumatoid arthritis- early onset RA
● Cause is unknown ● Tinnitus- ototoxic
● GI bleeding
Risk Factors

● Genetic predisposition
● Viral/ bacterial infections 2. DMARDs (Disease modifying antirheumatic drugs)
● Sulfasalazine
Pathophysiology
● Methotrexate (rheumatrex)
Autoimmune response→ ↑antibodies→↑antigen-antibody/ ● Chloroquine
immune complexes→ destroy synovium→ inflammation→
friction
3. Steroids oral
Manifestations

● Morning stiffness before for >30 minutes (+) pain when


stiff- initial sign
● Localized erythema, tenderness, and swelling
● Small joints are usually affected- wrists, metacarpal
● Systemic manifestations- anemia, fatigue, anorexia, OSTEOARTHRITIS
weight loss
● Migratory arthralgia- pain that travels from one joint to ● Non-inflammatory joint disease, degenerative
another ● Destruction of cartilage
● Symmetrical joint pain ○ Also manifests when a person is obese
● Chronic deformities ● Affectation is on the weight-bearing joints
○ Swan neck deformities(flexion of the distal ● Cause is unknown
interphalangeal joints (DIP) and hyperextension
of PIP) Risk Factors
○ Boutonniere deformities (Flexion of the proximal ● Old age >60 years old
interphalangeal joints (PIP)) ● Obesity
● Frequent trauma (falls)
● Strenuous jobs (those lifting heavy things, always standing)

Pathophysiology

Destruction of cartilage d/t (1) excessive weight bearing (2) wear and
tear

Manifestations
● Morning stiffness <30 min ● Pain tenderness (Dolor)
● Localized joint pain (arthralgia) ● Functio laesa (loss of function)
● Asymmetrical joint pain 3. Intermittent/ Intercritical
● Non-tender and cool to touch 4. Chronic gout (gradual)
● Fatigue (systemic sx) ● Alopurinol
● Chronic: ● (+) kidney stones composed of uric acids
○ Heberden’s Nodes- nodes in the DIP ● (+) tophi deposits contains uric acid crystals
○ Bouchard’s Nodes- nodes in the PIP
Diagnostic Procedure

● Serum uric acid


● Xray of the joint
● ESR and CRP

Management

● Acute gout- colchicine (anti inflammatory)


● SE:
○ Taken after meals d/t GI bleeding and upset
○ Diarrhea, once this appears drug should be
stopped
● Chronic gout- Allopurinol (uricosuric)
Diagnostics ○ excretes uric acid though xanthine oxidase inhibitor
○ Second is probenecid
● ESR, CRP- normal
○ SE: rashes
● xray of the joint
● NSAIDs except aspirin ASA (acetylsalicylic acid)
● Arthrocentesis- clear
SYSTEMIC LUPUS ERYTHEMATOSUS (SLE)
Management
● Autoimmune systemic disease
1. NSAIDs-aspirin
● Naproxen Risk Factors
● Taken PC- after meal
● Genetic predisposition
Paracetamol is not an NSAID. ● Family history
● Some medications cause pseudoSLE
2. COX2 inhibitors
● Celecoxib (Celebrex, Arcoxia) Pathophysiology
● Anti Inflammatory
Autoimmune response→ antigen→ antibodies (B cells)- ANA
● Advantage- does not cause GI bleeding, can be taken even
(Antinuclear Antibodies; only for SLE) or dsDNA→ damages all
without meals
nucleus of cells→ destruction of most cells
3. Steroids
● Anti Inflammatory Manifestations
● Route: Intraarticular on the affected joint
1. Skin- Erythematous rashes (initial manifestation)
GOUTY ARTHRITIS ● Malar rash/lupus
● Butterfly rash in the face
● Gout–hyperuricemia
● Alopecia
○ Inborn error of purine metabolism
● Avoid sun exposure
○ The liver convert proteins to uric acid
2. Renal- nephrotic syndrome (large proteins pass through
● Causes
the kidneys that causes damage)
○ High purine diet: internal organ meats, legumes,
3. CNS- neurosis (symptoms of stress (depression, anxiety,
proteins (x) eggs are not high in uric acid
obsessive behavior, hypochondria)), psychosis
○ Alcoholism
4. Hematology- anemia, leukopenia/agranulocytosis/
○ Malignancy
neutropenia, thrombocytopenia
○ Chemotherapy (Tumor lysis syndrome)
5. Skeletal- arthritis
■ Lysed cells release urea
○ Most common manifestation
○ Family history
6. Cardiovascular- pericarditis
● Monoarticular gout
○ Usually affects the metatarsal phalanges of the big Diagnostic
toe
1. Serum ANA
Stages 2. Serum dsDNA
3. IV pyelography (contrast medium dye is given to visualize
1. Asymptomatic
stones in the urinary tract)
● No manifestations, but (+) accumulating uric acid
4. ESR & CRP
○ Normal serum UA: >8 mg/dL
5. CT scan of the chest (pericarditis)
2. Acute gout
6. CBC
● Colchicine
● Erythema (rubor) Management
● Swelling (tumor)
● Warm to touch (Calor) 1. Steroids (DOC)
2. NSAIDs- to decrease dosage of steroids ● Purposes:
3. Chloroquine- antimalarial ○ To maintain body alignment
4. Immunosuppressants ○ Immobilization
○ Reduce the fracture (reduce- realignment)
Nursing Management ○ Decrease muscle spasm that will decrease pain
● Pain I. Types of Traction
1. Skin Traction
● Skin elastic bandages are used
RA OA Gouty SLE
● Short-term (<3 days)
Apply cold Weight Increased Avoid sun a. Dunlop- humerus fracture
(numbness) & reduction OFI: 2-3 L/day exposure b. Buck’s- hip, femur fracture
warm (Normal: 1.5 c. Russell’s- femur
compress Proper diet- L./day intake Apply d. Gallow’s/ Bryant’s- for children (<2 years old with
(increase less and output) sunblock fracture in the lower extremity)
blood flow d/t carbohydrates (highest SPF) i. Hips are elevated with a pillow with
vasodilation) and fats, high Avoid foam boots
q15 min protein; low alcoholic ● Countertraction- 10% of the body weight
caloric intake beverages ○ Sandbags or water bags are used
○ Should be freely hanging
Avoid purine
○ Countertractions are not used for children
foods
because their weights are too light
Elevate the ● Pin care is only for skeletal tractions
affected foot
Nursing Interventions
●Set up foot
cradle 1. Supine position
2. Do not remove the weight (countertraction)
● Except for intermittent traction to avoid irritation of
the skin
3. Maintain pulley systems unobstructed
4. Footboard to prevent foot drop (plantarflexion of the foot)
● This is a permanent contracture
5. Prevent external hip rotation
FRACTURES ○ Use of trochanter roll
○ If the head or neck of the femur is fractured→
● Any break in the continuity of the bones hip fracture
● Bones are hypervascular ○ Femoral head slipped from the acetabulum
Causes (socket)

● Trauma
● Vehicular accident
● Falls

Types

● Comminuted- splintered into pieces


● Simple/ closed fracture- (+) broken bone but has not
penetrated the skin
● Complex/ open fracture- (+) broken bone but has
penetrated the skin
● Pathologic- breaks d/t weakening of the bone
(osteoporosis)
● Greenstick- one side is broken then the other side is bent

Manifestations
2. Skeletal Traction
● Pain, tender to touch
● Guarding ● Surgical procedure that placed wires, pins, tongs, and
● Deformity- shortening of the bone d/t muscle spasms screws
● Ecchymosis (hematoma)/ contusion a. Halo traction- scoliosis, cervical spinal cord
● Decreased movement injury
● Crepitus- sound of grating bones b. Pelvic traction
● Compilation: hypovolemic shock c. 90-90 femur fracture

Diagnostic test

● Radiography (x-ray)

Management

1. Traction
Even without cast compartment syndrome may occur, the
fascia does not expand, blood will only accumulate in the
muscle layer→ compartment syndrome→ fasciotomy is
done
5. Avoid hair/blow drying the newly casted extremity (half-
cooked)

2. Open Reduction
d. Balanced Skeletal Traction ● FIxation (external and internal)
○ External- metallic device with pins; clean with
Nursing Interventions
chlorhexidine
● Provide pin care ○ Internal- (+) pins, rods, plates
○ Chlorhexidine to allow assessment d/t discoloration
Complications
caused by betadine
● Supine, flat in bed ● Hypovolemic shock
● Trapeze bar- to maintain independence of the patient ● Management:
○ Blood transfusion
○ Inotropes (Dobutamine and dopamine)
● Osteomyelitis
○ Purulent discharge form the bone
○ Warm to touch
○ Fever
○ Classic signs of inflammation
● Management:
○ Broad spectrum of IV antibiotics
○ Debridement
● Compartment syndrome
● Management:
○ Fasciotomy or bivalving
● DVT
II. Reduction ○ Homan’s sign
○ Management: anticoagulants (prevent venous
1. Closed Reduction- casting procedure blood clots)
● Plaster of Paris- dries for 24-48 hours (2 days) ● Fat embolism d/t long bone fracture (contains yellow bone
● Fiberglass- dries after 30 min marrow)
○ Contains lipids (fat globules are atherogenic–
● Plaster of paris can be used in complex platelets) that will cause pulmonary embolism
● Plastering is distal to proximal ○ Symptoms:
● Short cast- joints below the affected site is not included ■ Difficulty of breathing, shortness of
● Long cast- with joints; faster wound healing and breath
reduction ■ Increased RR, PR
■ Pleuritic chest pain- pain upon
inspiration
■ Thrombocytopenia
Nursing Care (petechiae/petechial rashes)
1. If newly casted, carry the extremities should be carried Management
using palms
● Position elevated using pillows ● Endotracheal tube connected to mechanical ventilation
2. Warm sensation inside the cast- normal (setting is hyperoxygenate that will cause respiratory
3. Hot spot- abnormal (infection) alkalosis)
● Normal cast upon palpation should be cool ● Heparin sodium
● If (+) infection, immediately report for antibiotics to ○ Check PTT everyday
prevent osteomyelitis ○ ↑PTT for 1.5-2 to determine is effective
4. WOF: compartment syndrome d/t too tight application of ○ N: 30-45 seconds
cast ○ Get the PTT of the patient then multiply by 1.5 or 2
● First manifestation (6 Ps): tingling sensation, (45-90 seconds) to check if the dose should be
numbness, paresthesia of the distal parts (toes- adjusted or continued
dorsalis pedis/ fingers) ○ If the result surpasses the therapeutic result
● Unrelieved pain immediately report and stop
● Paralysis ○ Antagonist- IV protamine sulfate
● Pallor ● Warfarin
● Pulselessness ○ Check PT everyday
● Poikilothermia ○ N: 10-12 seconds then multiply by 1.5 to 2 (15-24
● Management: bivalving seconds)
○ If the result surpasses the therapeutic result
immediately report and stop
○ If no PT, check INR (N:2-3 seconds, value is fixed ○ (1) start with the assistive device on the affected
not multiplied anymore) side, (2) opposite foot, (3) assistive device on the
○ Antagonist- IV Vitamin K unaffected side, (4) affected foot
○ Avoid green leafy vegetables (because this is ● Swing-to
high in vitamin K and this will antagonize the ○ Move to the level of the crutches
effect of the drug) ● Swing through
○ SE: bleeding ○ (1) Move the crutches (2) move both feet past the
level
Nursing Management

Acute pain When climbing upstarts- (1) unaffected leg (good leg) should be
raised first, no crutches (2) bad leg with the two crutches
● Assess VS, pain severity (7-10)
○ BP is high when (+) pain When climbing down- (1) two crutches, (2) affected leg, (3) good
○ Severe pain may result to neurogenic shock leg
● Position- accident scene- whatever the position you have
seen the patient should be retained, do not be realigned as Cane:
to not incur more fracture (splint the patient)-- recovery Going up: (1) good leg, (2) cane with the bad leg
position Going down: (1) cane with bad leg (2) good leg
● Elevate the affected extremity if simple fractures only to
“Good goes to heaven, bad goes to hell”
prevent bleeding
● Administer morphine sulfate IV
○ Check RR before administering morphine because
Nursing Care (crutches)
opioids are respiratory depressants
○ Bedside antagonist: Naloxone (Narcan) ● Axillary pad should be 2 inches below the axilla
● Apply cold compress on a casted extremity ○ To prevent compression of the brachial plexus
(group of nerves that supplies the hand, arm, and
Impaired physical mobility
forearm) and prevent crutch palsy (manifested
● Use of assistive devices- help the patient to get out of bed by weakness of the forearm)
○ Hand pad
Assistive Devices ■ Elbow flexion (30-45 degrees)
■ To prevent joint contracture
Cane
Walker
● Canes are used by the unaffected side e.g., fracture on
the right leg, cane will be on the left side ● Most stable
● Bad leg/ affected side is moved with the assistive ● Patient should maintain vision in front
device (reciprocal motion) ● With four legs
● 4 inches lateral to the toe
● 2 inches anterior Additional notes:
● 6 inches diagonal/ lateral to the toe Devices should be at the level of the waistline of the patient.
● Nursing Care: Ankylosing spondylitis- arthritis of the cervical bones.
1. Hold the cane with the hand opposite of the injury
(reciprocal motion)
2. Tip of the cane should be 6 inches lateral to the
foot of the unaffected side

Crutches

Types of Gait:

● 2-pt
○ (1) Right crutch with left leg, (2) left crutch with right
leg (reciprocal motion)
● 3-pt
○ (1) Both crutches (2) affected (3) unaffected
○ One lower extremity is affected
● 4-pt

You might also like